Download An elderly woman with COPD presents with a decreased level of

Survey
yes no Was this document useful for you?
   Thank you for your participation!

* Your assessment is very important for improving the workof artificial intelligence, which forms the content of this project

Document related concepts

Medical ethics wikipedia , lookup

Adherence (medicine) wikipedia , lookup

Patient safety wikipedia , lookup

Electronic prescribing wikipedia , lookup

List of medical mnemonics wikipedia , lookup

Transcript
An elderly woman with COPD presents with a decreased level of consciousness, cyanosis to
her face and neck, and labored respirations. Her pulse is rapid and weak and her oxygen
saturation is 76%. You should:!
!
A:!
apply oxygen via nasal cannula and reassess her respiratory status.!
B:!
insert a nasal airway and give her oxygen via a nonrebreathing mask.!
C:!
assist her ventilations with a bag-mask device and high-flow oxygen.!
D:!
avoid high-flow oxygen because this may cause her to stop breathing.!
The correct answer is C;!
!
Reason:!
!
The patient in this scenario is experiencing an exacerbation (worsening) of her COPD. Her
decreased level of consciousness; cyanosis; weak, rapid pulse; low oxygen saturation (SpO2);
and labored breathing clearly indicate that she is not breathing adequately. Therefore, you
should assist her ventilations with a bag-mask device and high-flow oxygen; if you don’t, she will
continue to deteriorate, possibly to the point of cardiac arrest. If needed, insert a nasal airway
adjunct to help keep her airway open. Regardless of the patient’s history of COPD, you must
NOT withhold oxygen from her. Respiratory depression in COPD patients who receive high-flow
oxygen is highly uncommon. Death due to hypoxia, however, is very common.!
!
==================================!
!
Prior to applying a nonrebreathing mask on a patient with difficulty breathing, you should:!
!
A:!
insert a nasopharyngeal airway to maintain airway patency.!
B:!
perform a complete exam to assess the degree of hypoxia.!
C:!
prefill the reservoir bag to ensure delivery of 100% oxygen.!
D:!
set the flow rate to no more than 10 liters per minute.!
The correct answer is C;!
!
Reason:!
!
After attaching the nonrebreathing mask to the oxygen source, the flowmeter should be set to
between 12 and 15 L/min. The reservoir bag is then prefilled with oxygen, which will allow the
delivery of high-flow oxygen. Unless the patient has a decreased level of consciousness, a
nasopharyngeal airway is not required before applying a nonrebreathing mask. The need for
supplemental oxygen should be determined early in your assessment; do not perform an indepth exam before deciding to administer oxygen.!
!
==================================!
!
While ventilating an apneic patient with a bag-mask device, you note minimal rise of the chest
each time you squeeze the bag. You should:!
!
A:!
B:!
C:!
D:!
evaluate the mask-to-face seal and the position of the patient's head.!
ensure that the reservoir is properly attached to the bag-mask device.!
suction the patient’s mouth for 15 seconds and reattempt ventilations.!
squeeze the bag harder to ensure delivery of adequate tidal volume.!
The correct answer is A;!
!
Reason:!
!
If the patient's chest rises minimally or not at all when you are ventilating him or her with the
bag-mask device, you should first reevaluate the mask-to-face seal and make sure that the
patient's head is properly positioned. The most common complication associated with bag-mask
ventilation is difficulty in maintaining an adequate mask-to-face seal. If repositioning the head
does not correct the problem, you should ensure that you are squeezing the bag hard enough to
deliver adequate tidal volume. Caution must be used, however, when ventilating a patient;
breaths that are delivered too forcefully or too fast (hyperventilation) may cause an increase in
intrathoracic pressure, thus impeding blood return to the heart and decreasing cardiac output.
Forceful ventilations may also cause significant gastric distention. Therefore, you should deliver
each breath over a period of one second—just enough to produce visible chest rise. The
patient's mouth should be suctioned only if it contains blood or other secretions.!
!
==================================!
!
An unresponsive apneic patient’s chest fails to rise after two ventilation attempts. You should:!
!
A:!
immediately proceed to chest compressions.!
B:!
reposition the head and reattempt to ventilate.!
C:!
attempt to ventilate again using more volume.!
D:!
suction the airway and reattempt ventilations.!
The correct answer is A;!
!
Reason:!
!
If your initial attempt to ventilate an apneic patient is unsuccessful (that is, you meet resistance
or the chest fails to visibly rise), reposition the patient’s head and reattempt to ventilate. If the
second ventilation is unsuccessful, you should proceed under the assumption that the patient
has a severe (complete) airway obstruction. Perform 30 chest compressions, open the airway,
and visualize the mouth (remove an object only if you can see it). If you are able to remove the
foreign object, attempt to ventilate. If you are not, continue chest compressions. Continue this
sequence until the obstruction is relieved or an advanced life support (ALS) ambulance arrives.
If ALS response will be delayed, transport, continuing your attempts to relieve the obstruction en
route, and coordinate a rendezvous with the ALS unit.!
!
==================================!
!
While managing a patient with acute shortness of breath, you attempt to apply a nonrebreathing
mask set at 12 L/min. The patient pulls the mask away from his face, stating that it is smothering
him. You should:!
!
A:!
increase the oxygen flow and reapply the mask.!
B:!
inform the patient that refusing oxygen may result in his death.!
C:!
reassure the patient and apply a nasal cannula instead.!
D:!
securely tape the oxygen mask to the patient’s face.!
The correct answer is C;!
!
Reason:!
!
Some adults cannot tolerate the oppressive feeling of an oxygen mask over their face; children
are typically less tolerant than adults. You should provide reassurance to the patient and apply a
nasal cannula at 2 to 6 L/min, which will likely be better tolerated. Do not force an oxygen mask
on a patient's face; doing so will only increase his or her anxiety, which will increase his or her
body's oxygen consumption and demand.!
!
==================================!
!
Which of the following signs or symptoms is indicative of cerebral hypoxia?!
!
A:!
Heart rate greater than 120 beats/min!
B:!
Decreased level of consciousness!
C:!
Diffuse wheezing on exhalation!
D:!
Chief complaint of dyspnea!
The correct answer is B;!
!
Reason:!
!
Dyspnea, a feeling of shortness of breath, is a symptom of a condition that can cause cerebral
hypoxia (eg, CHF, COPD); however, dyspnea itself does not indicate cerebral hypoxia.
Wheezing, a whistling sound that indicates bronchospasm, is a sign; like dyspnea, it indicates
the presence of a condition that can cause cerebral hypoxia (eg, asthma). Tachycardia can
occur for many reasons; cerebral hypoxia is but one. Of the choices listed, a decreased level of
consciousness is most indicative of cerebral hypoxia. As oxygen levels in the brain decrease
and carbon dioxide levels increase, the patient's mental status deteriorates.!
!
==================================!
!
Shallow respirations are an indication of:!
!
A:!
increased oxygen intake.!
B:!
increased carbon dioxide removal.!
C:!
increased minute volume.!
D:!
decreased tidal volume.!
The correct answer is D;!
!
Reason:!
!
Shallow respirations are an indication of decreased tidal volume. Tidal volume is the amount of
air (in milliliters [mL]) breathed into or out of the lungs in a single breath. Adequate tidal volume
is needed to bring in adequate amounts of oxygen and eliminate adequate amounts of carbon
dioxide. Patients with shallow breathing often need some form of positive-pressure ventilation
assistance (eg, bag-mask or pocket face mask device), especially if they have a decreased
mental status. Minute volume is the volume of air that is moved through the lungs per minute; it
is a product of tidal volume multiplied by the respiratory rate. If tidal volume is reduced, minute
volume will be reduced as well unless there is a compensatory increase in the respiratory rate. !
!
==================================!
!
After an initial attempt to ventilate an unresponsive apneic patient fails, you reposition the
patient's head and reattempt ventilation without success. You should next:!
!
A:!
turn the patient onto his side and deliver 5 to 10 back slaps.!
B:!
administer 5 to 10 abdominal thrusts and reattempt to ventilate.!
C:!
perform chest compressions, open the airway, and look in the mouth.!
D:!
perform continuous chest compressions until ALS personnel arrive.!
The correct answer is C;!
!
Reason:!
!
If you are unable to ventilate an unresponsive, apneic patient after two attempts, you should
assume that he or she has a severe (complete) foreign body airway obstruction. Immediately
perform 30 chest compressions (15 compressions if two EMTs are present and the patient is an
infant or child). Next, open the patient's airway and look inside the mouth. If you can see the
object, attempt to remove it with your finger (never perform blind finger sweeps of the mouth). If
you cannot see the object, continue chest compressions. If you are able to remove the object,
reattempt to ventilate. Unless paramedics are nearby, begin transport while continuing chest
compressions, opening the airway and looking in the mouth, and attempting to ventilate (if you
can remove the object). Abdominal thrusts are indicated for responsive children and adults with
a severe airway obstruction. Back slaps are indicated for a responsive infant with a severe
airway obstruction.!
!
==================================!
!
Which of the following airway sounds indicates a lower airway obstruction?!
!
A:!
Wheezing!
B:!
Stridor!
C:!
Gurgling!
D:!
Crowing!
The correct answer is A;!
!
Reason:!
!
Wheezing is a whistling sound that results from narrowing and/or inflammation of the
bronchioles in the lungs and is an indicator of a lower airway disease (ie, asthma, bronchiolitis).
Crowing and stridor are both high-pitched sounds that indicate an upper airway disease or
obstruction (ie, croup, epiglottitis, foreign body obstruction), and gurgling indicates secretions in
the oropharynx.!
!
==================================!
!
As you begin ventilating an unresponsive apneic man, you hear gurgling in his upper airway.
Your MOST immediate action should be to:!
!
A:!
suction the patient’s airway for no longer than 15 seconds.!
B:!
reposition the patient’s airway and continue ventilations.!
C:!
squeeze the bag-mask device with less force and reassess.!
D:!
quickly turn the patient onto his side so secretions can drain.!
The correct answer is D;!
!
Reason:!
!
Gurgling in the airway indicates the presence of vomitus or other secretions. If this is noted, you
should immediately turn the patient onto his side to allow the secretions to drain. After placing
the patient on his side, suction his airway for no longer than 15 seconds. To continue ventilating
a patient whose airway is full of vomitus or secretions will force the secretions into the trachea,
resulting in aspiration. Aspiration significantly increases mortality!!
!
==================================!
!
A 56-year-old man has labored, shallow breathing at a rate of 28 breaths/min. He is responsive
to pain only. You should:!
!
A:!
ventilate him with a bag-mask device at a rate of 30 breaths/min.!
B:!
suction his mouth for 15 seconds and insert an oropharyngeal airway.!
C:!
insert a nasopharyngeal airway and begin assisting his ventilations.!
D:!
place him on his side and administer oxygen via nonrebreathing mask.!
The correct answer is C;!
!
Reason:!
!
This patient in this scenario is not breathing adequately. He is responsive to pain only, and his
respirations are rapid, labored, and shallow. You should insert a nasopharyngeal airway, which
is usually well-tolerated in patients who are semiconscious and have a gag reflex, and assist his
ventilations with a bag-mask device. When assisting a patient's breathing, you should squeeze
the bag-mask device to ensure that he or she receives 10 to 12 adequate breaths per minute.
Do not hyperventilate the patient as this increases the risks of vomiting and aspiration.
Hyperventilation also increases intrathoracic pressure, which may impair venous return to the
heart (preload) and cause a decrease in cardiac output. Oxygen via nonrebreathing mask is
appropriate for patients who are breathing adequately, but are suspected of being hypoxic. The
recovery position (patient is placed on his or her side) is appropriate for unresponsive, uninjured
patients with adequate breathing.!
!
==================================!
!
Which of the following patients obviously needs positive-pressure ventilation assistance?!
!
A:!
Combative; respiratory rate of 24 breaths/min and deep!
B:!
Restless; respiratory rate of 12 breaths/min with adequate tidal volume!
C:!
Responsive to pain only; respiratory rate of 8 breaths/min and shallow!
D:!
Semiconscious; respiratory rate of 14 breaths/min and good chest rise!
The correct answer is C;!
!
Reason:!
!
Any patient with a decreased level of consciousness should be assessed for inadequate
breathing (eg, fast or slow respiratory rate, reduced tidal volume [shallow breathing]). Of the
patients listed, the patient who is responsive to pain only and has shallow respirations of 8
breaths/min clearly needs positive-pressure ventilation assistance. Slow, shallow respirations
will not produce the minute volume needed to support adequate oxygenation of the blood.!
!
==================================!
!
Snoring respirations in an unresponsive patient are usually the result of:!
!
A:!
swelling of the upper airway structures.!
B:!
foreign body airway obstruction.!
C:!
upper airway obstruction by the tongue.!
D:!
collapse of the trachea during breathing.!
The correct answer is C;!
!
Reason:!
!
In an unresponsive patient, the muscles of the tongue, which attach to the mandible, relax and
fall back over the posterior pharynx. This makes obstruction by the tongue the most common
cause of airway obstruction in the unresponsive patient. Foreign body upper airway obstructions
and upper airway swelling typically produce stridor, a high-pitched sound heard during
inhalation. Collapsing of the trachea during breathing would likely present with marked
respiratory distress.!
!
==================================!
!
A 50-year-old man, who fell approximately 20 feet and landed on a hard surface, is
semiconscious. You should:!
!
A:!
check for a carotid pulse if the patient is breathing rapidly.!
B:!
gently tilt the patient’s head back to assess for breathing.!
C:!
stabilize his head while performing the jaw-thrust maneuver.!
D:!
begin positive-pressure ventilations with a bag-mask device.!
The correct answer is C;!
!
Reason:!
!
Because of the significant mechanism of injury (fall of greater than 15 feet), spinal injury should
be assumed. The first step in managing this patient is to manually stabilize his head in a neutral
position and open his airway with the jaw-thrust maneuver, both of which can be performed
simultaneously. After the patient's airway is open, assess the rate and quality of his breathing
and treat accordingly. The head tilt-chin lift maneuver should not be used on a patient with a
possible spinal injury unless the jaw-thrust maneuver does not adequately open his or her
airway. The patient in this scenario is semiconscious; therefore, he has a pulse (pulseless
patients are unresponsive). If an uninjured patient is found to be unresponsive, you should
quickly assess for breathing by visualizing the chest. If the patient is not breathing or only has
agonal gasps, you should check for a carotid pulse.!
!
==================================!
!
Occasional, irregular breaths that may be observed in a cardiac arrest patient are called:!
!
A:!
ataxic respirations.!
B:!
Biot respirations.!
C:!
Cheyne-Stokes respirations.!
D:!
agonal gasps.!
The correct answer is D;!
!
Reason:!
!
Occasional, irregular breaths, called agonal gasps, may be observed in some patients shortly
after their heart stops beating. They occur when the respiratory center in the brain sends stray
signals to the respiratory muscles. Agonal gasps are not adequate because they are infrequent
and result in negligible tidal volume. Biot respirations are characterized by an irregular pattern,
rate, and depth of breathing with intermittent periods of apnea; they are commonly associated
with severe brain trauma. Ataxic respirations are ineffective, irregular breaths that may or may
not have an identifiable pattern; they are also commonly associated with severe brain trauma.
Cheyne-Stokes respirations are characterized by a crescendo-decrescendo pattern of breathing
with a period of apnea between each cycle (fast, slow, apnea). Cheyne-Stokes respirations may
occur in healthy people during certain phases of the sleep cycle; however, if they are grossly
exaggerated or occur in a patient with a head injury, they are an ominous sign.!
!
==================================!
!
Which of the following occurs during positive-pressure ventilation?!
!
A:!
The esophagus remains closed!
B:!
Blood is drawn back to the heart!
C:!
Intrathoracic pressure increases!
D:!
Oxygen is pulled into the lungs!
The correct answer is C;!
!
Reason:!
!
Negative-pressure ventilation, the act of normal breathing, occurs when the diaphragm and
intercostal muscles contract. The actions of these muscles create a vacuum (negative
pressure), which pulls oxygen-rich air into the lungs. Because of the negative pressure created
in the chest, blood is naturally drawn back to the heart. The esophagus remains closed during
normal breathing. In contrast, positive-pressure ventilation involves the forcing of air into the
lungs, such as what is provided during rescue breathing. Positive-pressure ventilation causes
an increase in intrathoracic pressure, which can impair blood flow back to the heart and cause a
decrease in cardiac output. During positive-pressure ventilation, the esophagus is forced open
and air enters the stomach (gastric distention); this could result in vomiting and aspiration.!
!
==================================!
!
A patient with a mild foreign body airway obstruction:!
!
A:!
has a low oxygen saturation.!
B:!
presents with a weak cough.!
C:!
is typically not cyanotic.!
D:!
has progressive difficulty breathing.!
The correct answer is C;!
!
Reason:!
!
Patients with a mild (partial) airway obstruction are able to move adequate amounts of air, but
will have varying degrees of respiratory distress. The patient can cough forcefully, although you
may hear wheezing in between coughs. Because the patient is able to move air effectively, the
level of oxygen in his or her blood remains adequate; therefore, cyanosis is typically absent. By
contrast, the patient with a severe (complete) airway obstruction cannot move air effectively and
cannot speak. If a cough is present, it is weak and ineffective. As the level of oxygen in the
blood falls, cyanosis develops, oxygen saturation falls, and the patient’s level of consciousness
decreases. A foreign body airway obstruction, mild or severe, is an acute event that presents
with an acute onset of difficulty breathing. Progressive (gradually worsening) difficulty breathing
is more consistent with diseases such as congestive heart failure and pneumonia.!
!
==================================!
!
Which of the following clinical findings is MOST consistent with a chronic respiratory disease?!
!
A:!
A barrel-shaped chest!
B:!
Use of accessory muscles!
C:!
An irregular pulse!
D:!
Altered mental status!
The correct answer is A;!
!
Reason:!
!
In certain lung diseases (eg, emphysema, asthma), air is gradually and continuously trapped in
the lungs in increasing amounts; this increases the anterior-posterior (front to back) diameter of
the chest, causing the chest to assume a barrel shape. A barrel-shaped chest indicates a
chronic respiratory disease. Accessory muscle use and an altered mental status in a patient with
respiratory distress should be assumed to be acute findings. An irregular pulse could be the
result of a primary cardiac problem, or a cardiac problem secondary to chronic hypoxemia in
patients with various respiratory diseases.!
!
==================================!
!
To obtain the MOST reliable assessment of a patient's tidal volume, you should:!
!
A:!
B:!
assess for retractions.!
count the respiratory rate.!
C:!
look at the rise of the chest.!
D:!
listen for airway noises.!
The correct answer is C;!
!
Reason:!
!
Tidal volume is the amount of air, in milliliters, breathed into or out of the lungs in a single
breath. The most effective (and practical) way to assess tidal volume is to evaluate the rise of
the patient’s chest. If the patient's chest rises minimally during inhalation, his or her respirations
are shallow; shallow respirations reflect a reduced tidal volume.!
!
==================================!
!
A 60-year-old woman presents with acute respiratory distress. She is conscious and alert, but
restless. Her respiratory rate is 26 breaths/min with adequate chest expansion, her breath
sounds are clear to auscultation bilaterally, and her oxygen saturation is 90%. Which of the
following is the MOST appropriate treatment for this patient?!
!
A:!
A nasopharyngeal airway and assisted ventilations!
B:!
A nasopharyngeal airway and supplemental oxygen!
C:!
A nasal cannula with the flowmeter set at 4 to 6 L/min!
D:!
Supplemental oxygen with a nonrebreathing mask!
The correct answer is D;!
!
Reason:!
!
Although the patient is restless—a sign of hypoxemia—she is conscious and alert and able to
maintain her own airway; therefore, an airway adjunct is not needed at this point. Furthermore,
her respirations, although increased in rate, are producing adequate tidal volume as evidenced
by adequate chest expansion. Therefore, she is not in need of assisted ventilation at this point.
Considering her oxygen saturation of 90%, the most appropriate treatment would be to
administer high-flow oxygen with a nonrebreathing mask and closely monitor her for signs of
inadequate breathing (ie, shallow breaths [reduced tidal volume], decreased level of
consciousness, cyanosis). An acutely hypoxemic patient requires more oxygen than a nasal
cannula can provide.!
!
==================================!
!
During your assessment of a trauma patient, you note massive facial injuries, weak radial
pulses, and clammy skin. What should be your MOST immediate concern?!
!
A:!
Internal bleeding and severe shock!
B:!
Applying 100% supplemental oxygen!
C:!
Providing rapid transport to a trauma center!
D:!
Potential obstruction of the airway!
The correct answer is D;!
!
Reason:!
!
Any trauma patient with severe maxillofacial trauma is at an extremely high risk of airway
compromise. The airway can be compromised by either mandibular fractures, in which the
tongue may occlude the airway, or severe oral bleeding, in which blood clots can obstruct the
airway. Correct ANY airway problems immediately upon discovery, ensure adequate ventilation
and oxygenation, assess for and treat other life-threatening injuries, and prepare for rapid
transport.!
!
==================================!
!
During your assessment of an unresponsive adult female, you determine that she is apneic. You
should:!
!
A:!
deliver two rescue breaths.!
B:!
assess for a carotid pulse.!
C:!
begin chest compressions.!
D:!
place an oropharyngeal airway.!
The correct answer is B;!
!
Reason:!
!
As soon as you determine that an adult patient is apneic or only has agonal gasps, you should
assess for a carotid pulse for at least 5 seconds but no more than 10 seconds. If the patient has
a pulse, provide rescue breathing at a rate of 10 to 12 breaths/min (one breath every 5 to 6
seconds). If the patient does not have a pulse, perform 30 chest compressions and then open
the airway and deliver 2 rescue breaths. When managing a patient who is in cardiac arrest, it is
critical to minimize interruptions in chest compressions and to avoid delays in starting chest
compressions. After starting CPR, apply the AED as soon as one is available. An airway adjunct
should also be inserted as soon as possible. !
!
==================================!
!
You receive a call for a 49-year-old woman who passed out. The patient's husband tells you that
they were watching TV when the incident occurred. No trauma was involved. The patient is
semiconscious and has cyanosis to her lips. After opening her airway with the head tilt-chin lift
maneuver, you should:!
!
A:!
insert an oropharyngeal airway.!
B:!
assess her respiratory effort.!
C:!
begin ventilation assistance.!
D:!
insert a nasopharyngeal airway.!
The correct answer is D;!
!
Reason:!
!
In the absence of trauma, open the patient’s airway with the head tilt-chin lift maneuver. To help
maintain airway patency, a nasopharyngeal airway should be inserted. Your patient is
semiconscious, not unconscious, so she will likely gag if you attempt to insert an oropharyngeal
airway; this may result in aspiration if she vomits. Remember, you must first open the patient’s
airway and, if needed, suction any secretions from the mouth. Next, insert an airway adjunct
and assess respiratory effort. The method of oxygenation you provide depends on the adequacy
of the patient's breathing.!
!
==================================!
!
Tidal volume is defined as the:!
!
A:!
volume of air inhaled or exhaled per breath.!
B:!
volume of air moved in and out of the lungs each minute.!
C:!
total volume of air that the lungs are capable of holding.!
D:!
volume of air that remains in the upper airway.!
The correct answer is A;!
!
Reason:!
!
Tidal volume (VT) is the amount of air that is inhaled or exhaled per breath; it is normally 500
mL in a healthy adult male. Tidal volume is assessed by noting the depth of a patient's
breathing. Shallow breathing, for example, indicates a reduced tidal volume. The volume of air
that remains in the upper respiratory tract (eg, larger bronchi, trachea) is called dead space
volume (VD); it is approximately 30% of the adult male's tidal volume and does not participate in
pulmonary gas exchange. The volume of air that moves in and out of the lungs each minute,
and does participate in pulmonary gas exchange, is called alveolar minute volume (VA). It is
calculated by multiplying the tidal volume (minus the dead space volume) and the respiratory
rate. Therefore, if an adult male has a tidal volume of 500 mL and a respiratory rate of 18
breaths/min, his alveolar minute volume is 6,300 mL (500 mL [VT] - 150 mL [VD] × 18 [breaths/
min] = 6,300 mL [VA]). The maximum volume of air that the lungs are capable of holding is
called the total lung capacity (TLC); it is approximately 6 L in the healthy adult male.!
!
==================================!
!
When ventilating an apneic patient, you note decreased ventilatory compliance. This means
that:!
!
A:!
the lungs are difficult to ventilate.!
B:!
you meet no resistance when ventilating.!
C:!
fluid is occupying the alveoli.!
D:!
the upper airway is blocked.!
The correct answer is A;!
!
Reason:!
!
As it applies to artificial ventilation, compliance is the ability of the lungs to expand during
ventilation. Increased ventilatory compliance means that no resistance is met when you
ventilate the patient; you can ventilate the lungs with ease. Decreased ventilatory compliance
means that significant resistance is met when you ventilate the patient; the lungs are difficult to
ventilate. Conditions such as upper airway obstruction, widespread bronchospasm, fluid in the
alveoli (eg, pulmonary edema), and COPD can all cause decreased ventilatory compliance.!
!
================================== !
!
The MOST appropriate treatment for a semiconscious patient with slow, shallow respirations
includes:!
!
A:!
an oropharyngeal airway and assisted ventilation with a bag-mask device.!
B:!
a nasopharyngeal airway and assisted ventilation with a bag-mask device.!
C:!
a nasopharyngeal airway and high-flow oxygen via a nonrebreathing mask.!
D:!
an oropharyngeal airway and high-flow oxygen via a nonrebreathing mask.!
The correct answer is B;!
!
Reason:!
!
Semiconscious patients are not fully able to protect their own airway and require an airway
adjunct. The nasopharyngeal airway is indicated for semiconscious patients because they often
have an intact gag reflex; the oropharyngeal airway is contraindicated in any patient with an
intact gag reflex. Slow, shallow respirations will not provide the minute volume needed to
support adequate oxygenation and should be treated with positive-pressure ventilation
assistance (eg, bag-mask device, pocket face mask).!
!
==================================!
!
A reduction in tidal volume would MOST likely result from:!
!
A:!
flaring of the nostrils.!
B:!
increased minute volume.!
C:!
unequal chest expansion.!
D:!
accessory muscle use.!
The correct answer is C;!
!
Reason:!
!
Unequal (asymmetrical) or minimal expansion of the chest results in a decrease in the amount
of air inhaled per breath (tidal volume). Accessory muscle use and nasal flaring are signs of
increased work of breathing, which represents an attempt to maintain adequate tidal volume
(and therefore, minute volume). An increase in tidal volume, respiratory rate, or both, would
result in an increase in minute volume. It should be noted, however, that a markedly fast
respiratory rate would cause a natural decrease in tidal volume. For example, a patient
breathing at a rate of 40 breaths/min would likely only inhale air into the anatomic dead space
before promptly exhaling it.!
!
==================================!
!
After an adult cardiac arrest patient has been intubated by a paramedic, you are providing
ventilations as your partner performs chest compressions. When ventilating the patient, you
should:!
!
A:!
B:!
C:!
deliver 2 breaths during a brief pause in chest compressions.!
deliver each breath over 2 seconds at a rate of 12 to 15 breaths/min.!
hyperventilate the patient to maximize carbon dioxide elimination.!
D:!
deliver each breath over 1 second at a rate of 8 to 10 breaths/min.!
The correct answer is D;!
!
Reason:!
!
When ventilating an adult cardiac arrest patient with an advanced airway in place (ie, ET tube,
multilumen airway, supraglottic airway), you should deliver each breath over a period of 1
second—just enough to produce visible chest rise—at a rate of 8 to 10 breaths/min (one breath
every 6 to 8 seconds). Do not attempt to synchronize ventilations with chest compressions once
the airway has been secured with an advanced device. Hyperventilation should be avoided as it
may result in increased intrathoracic pressure, decreased blood return to the heart, and as a
result, less effective chest compressions.!
!
==================================!
!
Which of the following is the preferred initial method for providing artificial ventilations to an
apneic adult?!
!
A:!
Two-person bag-valve-mask technique with 100% oxygen!
B:!
Mouth-to-mask technique with supplemental oxygen!
C:!
Flow-restricted, oxygen-powered ventilation device!
D:!
One-person bag-valve-mask technique with 100% oxygen!
The correct answer is B;!
!
Reason:!
!
The preferred initial method for providing artificial ventilations is the mouth-to-mask technique
with one-way valve and supplemental oxygen attached. Evidence has show that rescuers who
ventilate patients infrequently have difficulty maintaining an adequate seal with a bag-mask
device. Because both of the rescuer’s hands are freed up when using a pocket face mask, it is
easier to maintain an adequate seal, thus providing more effective ventilations. Of course, if two
rescuers are available to manage the airway, the two-person bag-mask device technique should
be used. The flow-restricted, oxygen-powered ventilation device, also referred to as the
manually-triggered ventilator or demand valve, requires an oxygen source to function and would
thus not be practical as an initial device for providing artificial ventilations.!
!
==================================!
!
You are dispatched to a residence for an elderly female who has possibly suffered a stroke. You
find her lying supine in her bed. She is semiconscious; has vomited; and has slow, irregular
breathing. You should:!
!
A:!
administer high-flow oxygen and place her on her side.!
B:!
perform a head tilt-chin lift and insert an oral airway.!
C:!
manually open her airway and suction her oropharynx.!
D:!
insert a nasal airway and begin assisting her breathing.!
The correct answer is C;!
!
Reason:!
!
This patient’s airway is in immediate jeopardy! The first step in caring for any semi- or
unconscious patient is to manually open the airway (eg, head tilt-chin lift, jaw-thrust) and ensure
it is clear of obstructions or secretions. Because the patient has vomited, she likely has vomitus
in her mouth, which must be removed with suction before she aspirates it into her lungs.
Mortality increases significantly if aspiration occurs. After opening her airway and removing any
vomitus or secretions from her oropharynx with suction, you should insert an airway adjunct (a
nasal airway in this case; the patient is semiconscious and likely has an intact gag reflex) and
begin assisting her breathing with a bag-mask device. Her respiratory effort is inadequate and
should be treated with some form of positive-pressure ventilation, not a nonrebreathing mask.
Placing a semi- or unconscious patient on his or her side (recovery position) is only appropriate
if he or she is breathing adequately; this patient is not.!
!
==================================!
!
The lower airway begins at the:!
!
A:!
cricoid cartilage.!
B:!
larynx.!
C:!
trachea.!
D:!
epiglottis.!
The correct answer is B;!
!
Reason:!
!
Anatomically, the lower airway begins at the larynx (voice box). The cricoid cartilage is a firm
cartilage ring that forms the inferior (lower) part of the larynx. The trachea is connected to the
larynx and extends downward to form the left and right mainstem bronchi. The epiglottis is an
upper airway structure; it is a leaf-shaped structure above the larynx that prevents food and
liquid from entering the trachea during swallowing.!
!
==================================!
!
A 60-year-old female is found unresponsive. She is cyanotic, is making a snoring sound while
she breathes, and has a slow respiratory rate. You should:!
!
A:!
ventilate her with a bag-mask device.!
B:!
insert an airway adjunct.!
C:!
suction her airway for 15 seconds.!
D:!
manually open her airway.!
The correct answer is D;!
!
Reason:!
!
Before you can assess and manage a patient’s breathing, you must ensure that his or her
airway is open first; this patient’s airway is not open! Snoring respirations indicate partial
blockage of the airway by the tongue. Manually open her airway, using the head tilt-chin lift or
jaw-thrust maneuver, and ensure that her airway is clear of secretions. If needed, suction her
oropharynx for up to 15 seconds. After manually opening her airway and removing any
secretions with suction, insert an airway adjunct (eg, oral or nasal airway) to assist in
maintaining airway patency. Slow respirations and cyanosis in an unresponsive patient are
obvious signs of inadequate breathing; assist the patient’s ventilations with a bag-mask device
and high-flow supplemental oxygen.!
!
==================================!
!
When ventilating an unresponsive apneic adult with a bag-mask device, you should ensure that:!
!
A:!
an airway adjunct has been inserted.!
B:!
ventilations occur at a rate of 20 breaths/min.!
C:!
the pop-off relief valve is manually occluded.!
D:!
you are positioned alongside the patient.!
The correct answer is A;!
!
Reason:!
!
When ventilating an unresponsive apneic patient with a bag-mask device, you should ensure
that an oral or nasal airway adjunct is inserted, which will keep the tongue off of the posterior
pharynx. When ventilating a patient with a bag-mask device, it is best for you to be positioned at
the patient's head to allow for better control of the head. Ventilations in the apneic adult with a
pulse (ie, not in cardiac arrest) should be provided at a rate of 10 to 12 breaths/min (one breath
every 5 to 6 seconds). Generally, only pediatric sized bag-mask devices have pop-off relief
valves, which should NOT be occluded, because they help prevent overinflation of the patient's
lungs.!
!
==================================!
!
Which of the following injuries or conditions should be managed FIRST?!
!
A:!
A large open abdominal wound!
B:!
Fluid drainage from both ears!
C:!
Bleeding within the oral cavity!
D:!
Bilateral fractures of the femurs!
The correct answer is C;!
!
Reason:!
!
Any injury or condition that jeopardizes the airway has priority over all else. If blood or other
secretions within the mouth are not suctioned immediately, aspiration may occur; this
significantly increases mortality. After securing a patent airway, control any external bleeding.
Ideally, you and your partner should treat airway problems and external bleeding at the same
time.!
!
==================================!
!
Which of the following is the MOST correct technique for ventilating an apneic adult who has a
pulse?!
!
A:!
Hyperventilate at a rate between 20 and 24 breaths/min.!
B:!
Ventilate at a rate of 15 breaths/min and look for visible chest rise.!
C:!
Deliver each breath over 1 second at a rate of 10 to 12 breaths/min.!
D:!
Deliver each breath over 1 second at a rate of 8 to 10 breaths/min.!
The correct answer is C;!
!
Reason:!
!
When ventilating an apneic adult who has a pulse, deliver each breath over a period of 1
second, at a rate of 10 to 12 breaths/min (one breath every 5 to 6 seconds), while observing for
visible chest rise. A ventilation rate of 8 to 10 breaths/min (one breath every 6 to 8 seconds) is
appropriate for infants (except newborns), children, and adult patients in cardiac arrest after an
advanced airway device (eg, ET tube, multilumen airway, supraglottic airway) has been
inserted. Do NOT hyperventilate the patient; doing so may impede blood return to the heart,
thus reducing cardiac output, secondary to hyperinflation of the lungs. Hyperventilation also
increases the incidence of gastric distention, regurgitation, and aspiration.!
!
==================================!
!
Which of the following yields the lowest minute volume?!
!
A:!
Respiratory rate of 14 breaths/min; tidal volume of 300 mL!
B:!
Respiratory rate of 10 breaths/min; tidal volume of 500 mL!
C:!
Respiratory rate of 16 breaths/min; tidal volume of 400 mL!
D:!
Respiratory rate of 12 breaths/min; tidal volume of 500 mL!
The correct answer is A;!
!
Reason:!
!
Minute volume is the amount of air moved through the lungs each minute, and is calculated by
multiplying tidal volume and respiratory rate. Therefore, a respiratory rate of 14 breaths/min and
a tidal volume of 300 mL would yield a minute volume of 4,200 mL (4.2 L), which is less than the
sum of any of the other values listed. Minute volume is affected by tidal volume, respiratory rate,
or both. An increase in tidal volume, respiratory rate, or both will cause an increase in minute
volume. A decrease in tidal volume, respiratory rate, or both will cause a decrease in minute
volume.!
!
==================================!
!
Signs of inadequate breathing in an unresponsive patient include:!
!
A:!
warm, moist skin.!
B:!
cyanotic oral mucosa.!
C:!
an irregular pulse.!
D:!
symmetrical chest rise.!
The correct answer is B;!
!
Reason:!
!
Signs of inadequate breathing in both responsive and unresponsive patients include a
respiratory rate that is too slow (less than 12 breaths/min) or too fast (greater than 20 breaths/
min); shallow (reduced tidal volume), irregular, or gasping respirations; asymmetrical (unequal)
chest rise; abnormal respiratory sounds, such as wheezing, stridor, or gurgling; and abnormal
skin color and condition (ie, cool or cold skin, pallor, diaphoresis, cyanosis). An irregular pulse
indicates a cardiac dysrhythmia.!
!
==================================!
!
Sonorous respirations are MOST rapidly corrected by:!
!
A:!
correctly positioning the head.!
B:!
suctioning the oropharynx.!
C:!
inserting an oropharyngeal airway.!
D:!
initiating assisted ventilations.!
The correct answer is A;!
!
Reason:!
!
Sonorous (snoring) respirations, which most commonly result from partial airway obstruction by
the tongue, are most rapidly corrected by simply positioning the head. This involves using either
the head tilt-chin lift or the jaw-thrust maneuver if trauma is suspected. To further ensure airway
patency, a simple adjunct (oral or nasal airway) may need to be inserted. The patient's airway
should be suctioned if a gurgling sound is heard during breathing.!
!
==================================!
!
The tidal volume of an unresponsive patient is rapidly assessed by:!
!
A:!
counting the patient's respiratory rate.!
B:!
auscultating his or her lung sounds.!
C:!
evaluating for the presence of cyanosis.!
D:!
observing for chest rise during inhalation.!
The correct answer is D;!
!
Reason:!
!
Tidal volume, a measure of the depth of breathing, is the amount of air in milliliters (mL) that is
moved into or out of the lungs during a single breath. The average tidal volume for an adult
male is approximately 500 mL. The quickest and most effective way to assess a patient’s tidal
volume is to observe his or her chest during breathing. If the chest rises adequately during
inhalation, tidal volume is probably adequate. If the chest rises very little, as with shallow
breathing, tidal volume is likely reduced. Auscultating breath sounds can give you an idea as to
the patient's tidal volume; bilaterally diminished breath sounds may indicate a reduced tidal
volume. It is quicker, however, to simply observe the chest for adequate rise. The presence of
cyanosis indicates hypoxemia and is not a direct reflection of tidal volume.!
!
==================================!
!
You are performing abdominal thrusts on a 19-year-old male with a severe airway obstruction
when he becomes unresponsive. After lowering him to the ground and placing him in a supine
position, you should:!
!
A:!
open his airway and look inside his mouth.!
B:!
assess for a carotid pulse for up to 10 seconds.!
C:!
begin CPR, starting with chest compressions.!
D:!
continue abdominal thrusts until ALS arrives.!
The correct answer is C;!
!
Reason:!
!
A patient with a severe airway obstruction may initially be responsive and then become
unresponsive during treatment. In this case, you know that an airway obstruction is the cause of
his or her problem. Therefore, after placing the patient in a supine position, you should begin
CPR, starting with chest compressions. Do not check for a pulse before starting chest
compressions. After performing 30 chest compressions (15 compressions in infants and children
when two EMTs are present), open the airway and look in the mouth. Only remove an object
that you can see; do not perform a blind finger sweep in any patient. If you cannot see the
object, resume chest compressions. Attempt to ventilate only if you retrieve an object from the
mouth.!
!
==================================!
!
A nonrebreathing mask is MOST appropriate to use on patients who:!
!
A:!
are breathing less than 12 times per minute.!
B:!
are semiconscious and breathing shallowly.!
C:!
are cyanotic and have a low oxygen saturation.!
D:!
have an adequate rate and depth of breathing.!
The correct answer is D;!
!
Reason:!
!
With the oxygen flow rate set at 15 L/min, the nonrebreathing mask can deliver an oxygen
concentration of 90% or greater. Unlike the bag-mask or pocket mask devices, which deliver
oxygen via positive pressure, the nonrebreathing mask delivers oxygen passively; therefore, the
patient must have an adequate rate and depth (tidal volume) of breathing in order to open the
one-way valve in the nonrebreathing mask and inhale oxygen from the reservoir bag. Shallow
(reduced tidal volume) breathing, bradypnea (slow breathing), cyanosis, a low oxygen
saturation, and a decreased level of consciousness are signs of inadequate breathing, and
should be treated with some form of positive-pressure ventilation assistance.!
!
==================================!
!
A young woman who overdosed on heroin is unresponsive with slow, shallow breathing. As you
attempt to insert an oropharyngeal airway, she begins to gag. You should:!
!
A:!
suction the patient’s oropharynx as you insert a nasopharyngeal airway.!
B:!
remove the oropharyngeal airway and be prepared to suction her mouth.!
C:!
make sure you are using the most appropriate size of oropharyngeal airway.!
D:!
place her on her side until she stops gagging and then suction her mouth.!
The correct answer is B;!
!
Reason:!
!
Although uncommon, an unresponsive patient may have an active gag reflex. If an
unresponsive patient begins to gag as you are attempting to insert an oropharyngeal airway,
you must remove the airway immediately and be prepared to suction if vomiting should occur.
Turn the patient on his or her side to facilitate drainage of secretions. Once the airway has been
cleared, a nasopharyngeal airway, which is better tolerated in patients with a gag reflex, should
be inserted.!
!
==================================!
!
An inaccurate pulse oximetry reading may be caused by:!
!
A:!
heat illnesses, such as heat stroke.!
B:!
excessive red blood cell production.!
C:!
a heart rate greater than 100 beats/min.!
D:!
severe peripheral vasoconstriction.!
The correct answer is D;!
!
Reason:!
!
A pulse oximeter measures the percentage of hemoglobin that is saturated with oxygen. Under
normal conditions, a patient's oxygen saturation (SpO2) ranges between 95% and 100% while
breathing room air. Although no definitive threshold for normal SpO2 values exists, an SpO2
that is less than 95% in a nonsmoker may indicate hypoxemia. Of the factors listed, several
peripheral vasoconstriction (ie, hypothermia, cigarette smoking, chronic hypoxia) would be the
most likely to produce an inaccurate SpO2 reading. When the peripheral vasculature constricts,
blood is shunted to the core of the body; in such cases, the pulse oximeter would likely yield a
falsely low reading (or no reading at all). Other factors that can cause inaccurate readings
include dark or metallic nail polish, dirty fingers, and abnormal hemoglobin binding (ie, carbon
monoxide [CO] poisoning). It is important to note that the pulse oximeter is designed to detect
gross abnormalities, not subtle changes, and should be used in conjunction with a thorough
clinical assessment of the patient.!
!
==================================!
!
Which of the following statements regarding the head tilt-chin lift maneuver is correct?!
!
A:!
It can only be used in conjunction with an oropharyngeal airway.!
B:!
It should be used in conjunction with an appropriate airway adjunct.!
C:!
It is the technique of choice for patients with potential spinal injury.!
D:!
It should be used on all unresponsive patients that you encounter.!
The correct answer is B;!
!
Reason:!
!
In an unresponsive patient without a suspected spinal injury, the head tilt-chin lift maneuver is
the recommended method for opening the airway. To aid in maintaining a patent airway, an
appropriate airway adjunct (ie, oral or nasal airway) should be used in conjunction with the head
tilt-chin lift maneuver. When inserted properly, the oral or nasal airway will keep the tongue off of
the posterior pharynx. You must remember that even once an airway adjunct has been placed,
proper positioning of the head must be maintained until the airway is secured more definitively
(ie, endotracheal intubation). If you suspect that the unresponsive patient has a spinal injury, the
jaw-thrust maneuver should be used; however, if the jaw-thrust maneuver does not adequately
open the patient's airway, the head tilt-chin lift maneuver should be used.!
!
==================================!
!
An unresponsive 60-year-old male is apneic and has a weak, rapid pulse. His oxygen saturation
reads 79%. You should:!
!
A:!
deliver one breath over 1 second every 5 to 6 seconds.!
B:!
mildly hyperventilate him until his oxygen saturation improves.!
C:!
ventilate at a rate of 8 to 10 breaths/min, ensuring visible chest rise.!
D:!
use a pocket face mask to deliver 12 to 20 breaths/min.!
The correct answer is A;!
!
Reason:!
!
When ventilating an apneic adult with a pulse, deliver one breath every 5 to 6 seconds (10 to 12
breaths/min). A ventilation rate of 12 to 20 breaths/min (one breath every 3 to 5 seconds) is
appropriate for infants and children. Regardless of the patient’s age or ventilation device you are
using (eg, bag-mask device, pocket face mask), each breath should be delivered over a period
of 1 second (enough to produce visible chest rise). Do not hyperventilate any patient, even
mildly, as this may cause a decrease in venous return to the heart secondary to hyperinflation of
the lungs. Hyperventilation also increases the risks of gastric distention, regurgitation, and
aspiration. After an advanced airway device has been inserted (eg, ET tube, multilumen airway,
supraglottic airway) in a cardiac arrest patient, you should no longer perform “cycles” of CPR;
the compressor delivers compressions at a rate of at least 100/min and the ventilator delivers 8
to 10 breaths/min (one breath every 6 to 8 seconds). This ventilatory rate during cardiac arrest
applies to all age groups, except the newborn.!
!
==================================!
!
Which of the following describes the correct technique for inserting a nasopharyngeal airway?!
!
A:!
Rotate the device as you insert it into the right nostril.!
B:!
Insert the device with the bevel facing the septum.!
C:!
Apply firm, gentle pressure if you meet resistance during insertion.!
D:!
Insert the device with the bevel facing the lateral part of the nose.!
The correct answer is B;!
!
Reason:!
!
Lubricate the nasopharyngeal airway with a water-soluble gel. Insert it into the larger nostril with
the curvature following the floor of the nose. If using the right nostril, the bevel should face the
septum. If using the left nostril, insert the device with the tip pointing upward, which will allow the
bevel to face the septum. Gently advance the airway. If using the left nostril, insert the device
until slight resistance is met, and then rotate it 180 degrees into position. This rotation is not
required if using the right nostril. Continue until the flange of the device rests against the nostril.
If you feel any resistance or obstruction, remove the device and insert it into the other nostril.
Forcing the airway into place may cause trauma to the nasal mucosa and unnecessary
bleeding, which the patient could potentially aspirate.!
!
==================================!
!
Which of the following would MOST likely occur if an adult patient is breathing at a rate of 45
breaths/min with shallow depth?!
!
A:!
The lungs would become hyperinflated, potentially causing a pneumothorax.!
B:!
Alveolar minute volume would increase due to the rapid respiratory rate.!
C:!
Most of his or her inhaled air will not go beyond the anatomic dead space.!
D:!
The volume of air that reaches the alveoli would increase significantly.!
The correct answer is C;!
!
Reason:!
!
Alveolar minute volume, the amount of air that reaches the alveoli per minute and participates in
pulmonary respiration, is affected by tidal volume, respiratory rate, or both. If the respiratory rate
decreases, tidal volume must increase in order to maintain adequate alveolar minute volume.
Conversely, if tidal volume decreases, the respiratory rate must increase accordingly. However,
if the respiratory rate is extremely fast, especially if the depth of breathing is shallow (reduced
tidal volume), most of the inhaled air will only make it to the anatomic dead space (ie, trachea,
larger bronchi) before it is promptly exhaled. As a result, alveolar minute volume would
decrease, resulting in inadequate pulmonary respiration and hypoxia. For this reason, patients
with rapid, shallow breathing often require ventilation assistance. Pulmonary hyperinflation
would not be an issue in a patient with exceedingly fast breathing and reduced tidal volume
because very little air is actually reaching the lungs.!
!
==================================!
!
Which of the following devices is contraindicated in patients with blunt chest trauma?!
!
A:!
Oxygen-powered ventilator!
B:!
Oral airway!
C:!
Bag-mask device!
D:!
Nasal airway!
The correct answer is A;!
!
Reason:!
!
The flow-restricted, oxygen-powered ventilation device (FROPVD), also referred to as an
oxygen-powered ventilator or manually-triggered ventilator, should not be used in patients with
chest trauma; it delivers oxygen under high pressure (40 L/min) and may worsen the patient’s
injury. The FROPVD is also associated with a high incidence of gastric distention. The FROPVD
is also contraindicated in pediatric patients and in patients with COPD. Infants and children have
small lungs; the high ventilatory pressure delivered by the FROPVD can easily cause a
pneumothorax. Patients with COPD often have air trapped in their lungs; excessive ventilatory
pressure may cause alveolar rupture or a pneumothorax.!
!
==================================!
!
How should you treat an unresponsive, uninjured patient with respirations of 16 breaths/min and
good chest expansion?!
!
A:!
Suctioning as needed and artificial ventilations!
B:!
Oropharyngeal suctioning and assisted ventilations!
C:!
Airway adjunct and oxygen via nonrebreathing mask!
D:!
Jaw-thrust maneuver and frequent suctioning!
The correct answer is C;!
!
Reason:!
!
After opening the airway of an unresponsive patient, an airway adjunct (oral or nasal airway)
should be inserted to keep the tongue from occluding the posterior pharynx. Oral and nasal
airways are used in conjunction with manual head positioning to help maintain a patent airway.
Unresponsive patients who are breathing adequately (good rate, adequate depth [tidal volume])
should receive high-flow oxygen via nonrebreathing mask. The patient must be monitored
closely for signs of inadequate breathing, which will require ventilatory assistance with a bagmask device. Suction the oropharynx only if blood or other secretions are in the patient's mouth.!
!
==================================!
!
A 22-year-old man crashed his motorcycle into a tree. He is found approximately 20 feet away
from his bike and is responsive to pain only. He is not wearing a helmet. You are unable to
effectively open his airway with the jaw-thrust maneuver. What should you do?!
!
A:!
Suction his oropharynx and reattempt the jaw-thrust.!
B:!
Apply high-flow oxygen and assess his carotid pulse.!
C:!
Insert an oral airway and assess his breathing.!
D:!
Carefully tilt his head back and lift up on his chin.!
The correct answer is D;!
!
Reason:!
!
Regardless of the situation, you MUST be able to establish and maintain a patent airway.
Without a patent airway, the patient will die. If you are unable to effectively open a trauma
patient’s airway with the jaw-thrust maneuver, you should carefully perform the head tilt-chin lift
maneuver. You cannot assess, not to mention treat, a patient’s airway if it is not open and clear
of secretions or foreign bodies.!
!
==================================!
!
You are administering oxygen at 15 L/min to a patient with respiratory distress. If you are using
a D cylinder (cylinder constant, 0.16), which reads 1,500 psi, how long will it take before you
need to replace the oxygen cylinder?!
!
A:!
9 minutes!
B:!
14 minutes!
C:!
18 minutes!
D:!
11 minutes!
The correct answer is B;!
!
Reason:!
!
The length of time you can use an oxygen cylinder depends on the type of cylinder you are
using, the pressure in the cylinder, and the oxygen flow rate. A D cylinder is a small oxygen
cylinder that is usually carried in the jump kit to the patient; it has a cylinder constant of 0.16.
The following method can be used to calculate cylinder duration: gauge pressure (in psi) – the
safe residual pressure (200 psi) × the cylinder constant ÷ flow rate in L/min. Using this formula,
your D cylinder will become depleted in about 14 minutes, as follows: 1,500 (psi) – 200 (safe
residual pressure) × 0.16 (cylinder constant) ÷ 15 (flow rate in L/min) = 13.86 (14 minutes). A full
oxygen cylinder should contain 2,000 psi. The safe residual pressure is the lowest acceptable
cylinder pressure before it should be replaced; it is usually 200 psi, although some EMS
systems use 500 psi as a safe residual pressure. Although you will switch to your on-board
oxygen (M cylinder) source when you load the patient into the ambulance, you should always
have at least one back-up portable cylinder (preferably two) when administering oxygen to a
patient at the scene, especially if you are giving high-flow (12 to 15 L/min) oxygen and/or your
on-scene time will be delayed (eg, lengthy extrication, moving a patient from the second floor,
etc).!
!
==================================!
!
Prior to your arrival, a woman experiencing an asthma attack took two puffs from her prescribed
inhaler without relief. After administering supplemental oxygen, you should:!
!
A:!
contact medical control for further advice.!
B:!
perform a detailed secondary assessment.!
C:!
administer one more puff from her inhaler.!
D:!
provide immediate transport to the hospital.!
The correct answer is A;!
!
Reason:!
!
Before assisting a patient with any medication other than oxygen, the EMT must ensure that the
medication is prescribed to the patient and then obtain authorization from medical control. In this
case, the physician probably will allow you to help the patient take one more puff from her
inhaler. Generally, up to three puffs from an inhaler are delivered in the field. It is important for
you to ask the patient how many puffs were taken from the inhaler before you arrived. The EMT
must correct any airway and/or breathing problems as quickly as possible. After doing so, a
secondary assessment can be performed.!
!
==================================!
!
The active, muscular part of breathing is called:!
!
A:!
inhalation.!
B:!
respiration.!
C:!
expiration.!
D:!
ventilation.!
The correct answer is A;!
!
Reason:!
!
The active, muscular part of breathing is called inhalation (inspiration). During inhalation, the
diaphragm and intercostal muscles contract. When the diaphragm contracts, it descends and
enlarges the thoracic cage from top to bottom. When the intercostal muscles contract, they lift
the ribs up and out. As the thoracic cage expands, the air pressure within the thorax decreases,
creating a slight vacuum. This pulls air through the trachea, causing the lungs to fill. Exhalation
(expiration) does not require muscular effort; it is a passive process. During exhalation, the
diaphragm and intercostal muscles relax. In response, the thorax decreases in size, and the ribs
and muscles assume a normal resting position. When the size of the thoracic cage decreases,
air in the lungs is compressed into a smaller space. The air pressure within the thorax then
becomes higher than the pressure outside and air is pushed out through the trachea.
Respiration is defined as the exchange of gases between the body and its environment.
Ventilation is defined as the movement of air into and out of the lungs.!
!
==================================!
!
When suctioning copious secretions from a semiconscious adult’s airway, you should:!
!
A:!
use a flexible catheter because it will remove the secretions faster.!
B:!
suction for up to 20 seconds while withdrawing the catheter.!
C:!
avoid touching the back of the airway with the suction catheter.!
The correct answer is C;!
!
Reason:!
!
When suctioning a patient’s airway, especially if he or she is semiconscious, you should avoid
touching the back of the airway with the suction catheter. Inserting the catheter too far may
stimulate the gag reflex, cause vomiting, and increase the risk of aspiration. Rigid (tonsil-tip)
catheters are best for removing large amounts of fluid from the airway. Flexible (whistle-tip)
catheters are used in situations in which rigid catheters cannot be used, such as with a patient
who has a stoma, patients whose teeth are clenched, or if suctioning the nose is necessary.
Apply suction while you are withdrawing the catheter. In the adult, suction for no longer than 15
seconds (10 seconds in children, 5 seconds in infants); suction not only removes secretions
from the airway, it also removes oxygen.!
!
==================================!
!
At the peak of the inspiratory phase, the alveoli in the lungs contain:!
!
A:!
more oxygen than carbon dioxide.!
B:!
high quantities of carbon dioxide.!
C:!
equal levels of oxygen and carbon dioxide.!
D:!
minimal levels of oxygen and carbon dioxide.!
The correct answer is A;!
!
Reason:!
!
At the peak of the inspiratory (inhalation) phase, the alveoli are filled with fresh oxygen that the
patient just breathed in. During the expiratory (exhalation) phase, the oxygen moves from the
alveoli to the left side of the heart and the carbon dioxide is exhaled into the atmosphere. The
process of oxygen and carbon dioxide exchange in the lungs is called pulmonary (external)
respiration.!
!
==================================!
!
A young female experienced massive facial trauma and is unresponsive. After several attempts,
you are unable to adequately open her airway with the jaw-thrust maneuver. You should:!
!
A:!
carefully tilt her head back and lift up on her chin.!
B:!
begin assisting her ventilations with a bag-mask device.!
C:!
apply oxygen and reattempt the jaw-thrust maneuver.!
D:!
insert a nasopharyngeal airway and apply oxygen.!
The correct answer is A;!
!
Reason:!
!
The jaw-thrust maneuver should be used to open the airway of any trauma patient because it
does not require manipulation of the neck. However, if the jaw-thrust maneuver does not
adequately open the patient’s airway, you should carefully perform the head tilt-chin lift
maneuver. The patient’s airway must be patent, regardless of the situation. After opening an
unresponsive patient’s airway, ensure that it is clear of secretions (suction as needed), insert an
airway adjunct, and assess breathing. If the patient is breathing adequately, administer high-flow
oxygen via a nonrebreathing mask. If the patient is breathing inadequately, assist his or her
ventilations with a bag-mask or pocket face mask device. You should avoid the use of a
nasopharyngeal airway in patients with massive head or facial trauma. If the airway is
accidentally pushed through a hole caused by the fracture, it may penetrate the cranium and
enter the brain.!
!
==================================!
!
The method by which you administer supplemental oxygen to a hypoxic patient depends
MOSTLY on the:!
!
A:!
patient’s level of consciousness and heart rate.!
B:!
severity of hypoxia and adequacy of breathing.!
C:!
presence or absence of cyanosis.!
D:!
suspected underlying cause of the hypoxia.!
The correct answer is B;!
!
Reason:!
!
All hypoxic patients, whatever the cause of their condition, should be treated with high-flow
oxygen. The method of oxygen delivery depends on the severity of the hypoxia and the
adequacy of the patient’s breathing. For example, a hypoxic patient who is breathing adequately
(eg, normal rate, adequate tidal volume) should receive oxygen via nonrebreathing mask.
However, if the patient is breathing inadequately (eg, fast or slow rate, shallow breathing
[reduced tidal volume]), he or she may require ventilation assistance with a bag-mask device.
The absence of cyanosis does not rule out hypoxia; cyanosis is a later sign and indicates
significant hemoglobin desaturation. A patient’s level of consciousness and heart rate can give
you clues as to the severity of his or her hypoxia; a decreased level of consciousness and a
rapid, weak pulse rate are signs of significant hypoxia.!
!
==================================!
!
A patient who is breathing with reduced tidal volume would MOST likely have:!
!
A:!
a respiratory rate of 14 breaths/min.!
B:!
shallow respirations.!
C:!
warm, moist skin.!
D:!
a prolonged inhalation phase.!
The correct answer is B;!
!
Reason:!
!
Tidal volume, a measure of the depth of breathing, is the amount of air (in milliliters [mL]) that is
moved into or out of the lungs during a single breath; in the average adult male, this is about
500 mL. Tidal volume cannot be quantified (that is, it cannot be assigned a numeric value) by
the EMT; however, it can be estimated by observing the adequacy of chest rise during
inhalation. A patient who is breathing with reduced tidal volume will have a shallow depth of
breathing; his or her chest rises minimally during inhalation. If a patient is not breathing with
adequate tidal volume, he or she will eventually become hypoxemic, which will cause the skin to
become cool and clammy and pale or cyanotic. Conversely, a patient with a prolonged
inhalation phase (eg, taking a deep breath) would experience an increase in tidal volume.
Minute volume is the amount of air moved through the lungs each minute; it is calculated by
multiplying tidal volume and respiratory rate. A respiratory rate of 14 breaths/min with adequate
tidal volume would result in adequate minute volume. Minute volume is affected by tidal volume,
respiratory rate, or both.!
!
==================================!
!
While providing initial ventilations to an apneic adult with a bag-mask device, you note minimal
rise of the chest despite an adequate mask-to-face seal. You should:!
!
A:!
attach an oxygen reservoir to the bag-mask device.!
B:!
suction the airway for up to 15 seconds.!
C:!
switch to a smaller mask for the bag-mask device.!
D:!
increase the volume of your ventilations.!
The correct answer is D;!
!
Reason:!
!
You must deliver adequate tidal volume to the patient to cause sufficient chest rise. If initial
ventilations cause minimal rise of the patient’s chest despite an adequate mask-to-face seal,
you should increase the volume of ventilations by squeezing the bag harder until the chest rises
adequately. Squeeze the bag with just enough force to cause adequate chest rise. Do not
routinely suction the patient's airway unless there are secretions in the mouth, as evidenced by
a gurgling sound during ventilations. Switching to a smaller mask would likely be ineffective as
air would probably leak from around the mask. Attach the oxygen reservoir to the bag-mask
device as soon as possible; although this does not influence the volume of air delivered to the
patient, it does allow you to deliver a higher concentration of oxygen.!
!
==================================!
!
A 40-year-old man is conscious, but has an increased rate of breathing. You should:!
!
A:!
assist his ventilations with a bag-mask device.!
B:!
immediately insert a nasopharyngeal airway.!
C:!
assess the regularity and quality of breathing.!
D:!
apply 100% oxygen via nonrebreathing mask.!
The correct answer is C;!
!
Reason:!
!
You must assess ALL parameters of a patient's breathing—rate, regularity, depth, and quality. If
a patient is breathing outside the normal ranges for his or her age, you should assess the depth,
quality, and regularity of the respirations in order to determine overall breathing adequacy. On
the basis of this assessment, the most appropriate treatment can be provided—oxygen via a
nonrebreathing mask or some form of positive-pressure ventilation (ie, bag-mask ventilations).
Breathing adequacy is not determined by respiratory rate alone. Conscious patients do not
require an artificial airway adjunct (ie, oral or nasal airway).!
!
==================================!
!
The MOST effective way to determine if you are providing adequate volume during artificial
ventilation is:!
!
A:!
assessing the chest for adequate rise.!
B:!
checking the skin for improvement of cyanosis.!
C:!
checking the pupils for increased reactivity.!
D:!
assessing the pulse for an improving heart rate.!
The correct answer is A;!
!
Reason:!
!
The goal of providing artificial ventilation is to provide adequate tidal volume to the patient so
that enough oxygen is delivered to the lungs, and ultimately, the cells of the body. The most
effective way to determine if adequate tidal volume is being delivered is to watch for the chest to
rise during each ventilation. Other signs of adequate artificial ventilation include improvement in
skin color, the return of the heart rate to a normal range, and ensuring that you are ventilating
the patient at the appropriate rate. If the adult is apneic but has a pulse, provide 10 to 12
breaths/min. If the adult is apneic and pulseless, provide 8 to 10 breaths/min after an advanced
airway device (ie, ET tube, multilumen airway, supraglottic airway) has been inserted.!
!
==================================!
!
A 60-year-old woman is experiencing severe respiratory distress. When you ask her a question,
she can only say two words at a time. Treatment for her should include:!
!
A:!
insertion of a nasopharyngeal airway.!
B:!
applying a nasal cannula set at 2 to 6 L/min.!
C:!
applying a nonrebreathing mask set at 15 L/min.!
D:!
assisted ventilation with a bag-mask device.!
The correct answer is D;!
!
Reason:!
!
Because the patient is only able to speak in minimal word sentences (two-word dyspnea) and is
experiencing severe respiratory distress, it is unlikely that she is ventilating adequately.
Therefore, you should assist her ventilations with a bag-mask device. If her breathing continues
as it is, she will become increasingly hypoxic and may lose consciousness. Because this patient
is conscious, you must explain to her that every time she takes in a breath, the bag-mask device
will be squeezed so that an adequate volume of air can be delivered. Clearly, this can cause the
patient great anxiety, so your reassurance during this procedure is important. If the patient will
not tolerate your attempts to assist her ventilations, apply a nonrebreathing mask and monitor
her closely.!
!
==================================!
!
Agonal respirations are not adequate because they are:!
!
A:!
infrequent, gasping respiratory efforts.!
B:!
characterized by a rapid, irregular pattern.!
C:!
associated with a prolonged inhalation phase.!
D:!
the result of an increase in tidal volume.!
The correct answer is A;!
!
Reason:!
!
A patient may appear to be breathing after his or her heart has stopped. These occasional,
gasping breaths are called agonal respirations (also called agonal gasps) and occur when the
respiratory centers in the brain continues to send signals to the respiratory muscles. Agonal
respirations are not adequate because they are infrequent, gasping respiratory efforts that
produce very little, if any, tidal volume. Patients with agonal respirations require artificial
ventilation.!
!
==================================!
!
Ventilation is defined as the:!
!
A:!
movement of air into and out of the lungs.!
B:!
exchange of oxygen and carbon dioxide at the cell level.!
C:!
elimination of carbon dioxide from the body.!
D:!
volume of air inhaled into the lungs in a single breath.!
The correct answer is A;!
!
Reason:!
!
Ventilation is defined as the movement of air into and out of the lungs. During negative-pressure
ventilation (normal breathing), the diaphragm and intercostal muscles contract, which increases
the vertical and horizontal dimensions of the chest cavity. As a result, a vacuum is created in the
chest and air is drawn into the lungs. Positive-pressure ventilation is the act of forcing air into
the lungs (ie, bag-mask ventilation). The volume of air inhaled or exhaled in a single breath is
called tidal volume. The exchange of gases between the body and its environment is called
respiration; therefore, the exchange of oxygen and carbon dioxide at the cell level is called
cellular (internal) respiration. During pulmonary (external) respiration, oxygen and carbon
dioxide are exchanged in the lungs; oxygenated blood returns to the left side of the heart and
carbon dioxide is eliminated from the body during exhalation.!
!
==================================!
!
The process of loading oxygen molecules onto hemoglobin molecules in the bloodstream is
called:!
!
A:!
diffusion.!
B:!
oxygenation.!
C:!
ventilation.!
D:!
respiration.!
The correct answer is B;!
!
Reason:!
!
Oxygenation is the process of loading oxygen molecules onto hemoglobin molecules in the
blood. Adequate oxygenation is required for internal (cellular) respiration to take place. Diffusion
is the process in which gases (oxygen and carbon dioxide) move from an area of higher
concentration to an area of lower concentration. Ventilation is the act of moving air into and out
of the lungs. Negative-pressure ventilation is the act of normal, unassisted breathing, and
occurs when the diaphragm and intercostal muscles contract, which creates a vacuum and
draws air into the lungs. Positive-pressure ventilation is the act of forcing air into the lungs, such
as when you are providing rescue breathing to an apneic patient or assisting the ventilations of
a patient who is breathing inadequately. Respiration is the exchange of gases between the body
and its environment. Pulmonary (external) respiration occurs when gases are exchanged in the
lungs and cellular (internal) respiration occurs when gases are exchanged at the cellular level.!
!
==================================!
!
A patient's skin will MOST likely become cyanotic if he or she has:!
!
A:!
an increase in the amount of arterial oxygen.!
B:!
a decrease in the amount of carbon dioxide.!
C:!
an overall increase in circulating red blood cells.!
D:!
a decrease in the amount of arterial oxygen.!
The correct answer is D;!
!
Reason:!
!
Cyanosis, a blue or purple tint to the skin, reflects an inadequate amount of oxygen in the
arterial blood. More specifically, cyanosis indicates that a significant amount of hemoglobin has
separated from the red blood cells (desaturation), which makes the arterial blood less able to
carry oxygen. An overall increase in the number of circulating red blood cells (polycythemia),
would likely cause a patient's skin to remain pink, not become cyanotic. Patients with cyanosis
must be given high-flow oxygen and, if needed, positive-pressure ventilations if they are apneic
or breathing inadequately (eg, fast or slow rate, shallow breathing [reduced tidal volume]).!
!
==================================!
!
Clinically, reduced tidal volume would MOST likely present with respirations that are:!
!
A:!
shallow.!
B:!
deep.!
C:!
slow.!
D:!
eupneic.!
The correct answer is A;!
!
Reason:!
!
Tidal volume is the amount of air, in milliliters, that is breathed into or out of the lungs in a single
breath. Shallow respirations (minimal chest rise) indicates that negative-pressure ventilation,
and therefore tidal volume, is inadequate. Deep respirations (hyperpnea) would cause an
increase in tidal volume. Slow respirations, especially if accompanied by a shallow depth of
breathing, would lead to a reduction in minute volume. Eupnea is the medical term for normal
breathing; therefore, eupneic respirations are of adequate rate, depth, and regularity.!
!
==================================!
!
Medications such as albuterol (Ventolin) relieve respiratory distress by:!
!
A:!
B:!
C:!
constricting the bronchioles in the lungs.!
contracting the smaller airways in the lungs.!
dilating the large mainstem bronchi of the airway.!
D:!
relaxing the smooth muscle of the bronchioles.!
The correct answer is D;!
!
Reason:!
!
Medications such as albuterol (Ventolin) and metaproterenol (Alupent) are in a class of drugs
called bronchodilators. They relax the smooth muscle found within the bronchioles in the lungs,
which causes them to dilate. This effect opens the air passages and improves the patient’s
ability to breathe.!
!
==================================!
!
You are assessing a middle-aged male who is experiencing respiratory distress. The patient has
a history of emphysema and hypertension. He appears fatigued; has weak retractions; and
labored, shallow breathing. Your MOST immediate action should be to:!
!
A:!
assess his oxygen saturation with a pulse oximeter.!
B:!
administer oxygen with a nonrebreathing mask.!
C:!
auscultate his breath sounds to detect wheezing.!
D:!
assist his ventilations with a bag-mask device.!
The correct answer is D;!
!
Reason:!
!
Your patient is NOT breathing adequately. He is fatigued; has weak retractions; and labored,
shallow breathing. If you do not treat him immediately, he may stop breathing altogether. You
should begin assisting his ventilations with a bag-mask device and high-flow oxygen. After
initiating ventilatory assistance, attach the pulse oximeter to assess his oxygen saturation and
auscultate his breath sounds. A nonrebreathing mask is appropriate for patients with difficulty
breathing who are moving air adequately; this patient is not!!
!
==================================!
!
During the inhalation phase of breathing:!
!
A:!
air passively enters the lungs as pressure within the thorax increases.!
B:!
the diaphragm and intercostal muscles contract and ascend.!
C:!
the muscles in between the ribs relax, which lifts the ribs up and out.!
D:!
pressure within the thorax decreases and air is drawn into the lungs.!
The correct answer is D;!
!
Reason:!
!
Inhalation is the active, muscular part of breathing. During inhalation, the diaphragm and
intercostal muscles contract. When the diaphragm contracts, it moves down (descends) slightly
and enlarges the thoracic cage from top to bottom. Contraction of the intercostal muscles, the
muscles in between the ribs, causes the ribs to move up and out. As we inhale, the combined
actions of these structures enlarge the thorax in all directions. The air pressure outside the body,
called the atmospheric pressure, is normally higher than the air pressure within the thorax. As
we inhale and the thoracic cage expands, the air pressure within the thorax decreases, creating
a slight vacuum. This draws air in through the trachea and into the lungs, a process called
negative-pressure ventilation.!
!
==================================!
!
In which of the following situations would you MOST likely encounter agonal gasps?!
!
A:!
Significant hypoxemia, regardless of the cause!
B:!
Any patient who is unresponsive due to hypoxia!
C:!
Shortly after becoming unresponsive and pulseless!
D:!
Occlusion of the posterior pharynx by the tongue!
The correct answer is C;!
!
Reason:!
!
Agonal gasps are occasional, irregular, and ineffective breaths. They are commonly observed in
patients shortly after they become unresponsive and pulseless (cardiac arrest). Agonal gasps
may also be observed in patients with a severe brain injury or cerebral anoxia (complete
absence of oxygen). Patients with agonal gasps require some form of positive-pressure
ventilation. Hypoxemic and hypoxic patients typically present with tachypnea (increased
respirations) in an attempt to eliminate carbon dioxide and bring in more oxygen. However, as
the hypoxic patient begins to decompensate, his or her respirations often become slow
(bradypnea). If the tongue is occluding the posterior pharynx, a characteristic snoring sound is
typically heard.!
!
==================================!
!
A 22-year-old male has a shard of glass impaled in his cheek. You look inside his mouth and
see minor bleeding. The patient is conscious and alert with adequate breathing. You should:!
!
A:!
carefully remove the shard of glass in the same direction that it entered.!
B:!
remove the shard of glass and place gauze in his mouth to control the bleeding.!
C:!
be prepared for severe bleeding as you carefully remove the shard of glass.!
D:!
carefully stabilize the shard of glass and allow him to suction his own mouth.!
The correct answer is D;!
!
Reason:!
!
It remains true that you should remove an impaled object if it compromises the airway or
impedes your ability to manage the airway. However, neither is the case with this patient
because he has an adequate airway. He is conscious and alert and has only minor bleeding in
his mouth. The safest approach, and most practical given the situation, would be to carefully
stabilize the shard of glass in place; consider wrapping the exposed glass with gauze to protect
yourself from getting cut. Since the patient is conscious and alert and has only minor oral
bleeding, it would not be unreasonable to hand him the suction catheter and allow him to use it
as needed. Be sure to instruct the patient to use the suction and not to swallow any blood. Keep
in mind that if you attempt to remove the shard of glass, you risk cutting yourself and causing
further injury to the patient.!
!
==================================!
!
A properly placed oropharyngeal airway:!
!
A:!
eliminates the need to perform a head tilt-chin lift.!
B:!
will not stimulate a conscious patient’s gag reflex.!
C:!
keeps the tongue off of the posterior pharynx.!
D:!
prevents aspiration if the patient regurgitates.!
The correct answer is C;!
!
Reason:!
!
The oropharyngeal (oral) airway is an artificial adjunct used to keep the tongue away from the
posterior pharynx (back of the throat), thus preventing it from blocking the upper airway. It is
used in conjunction with, not in lieu of, the head tilt-chin lift or jaw-thrust maneuver to maintain
patency of the airway. The oral airway will not prevent aspiration if the patient regurgitates
because it does not occlude the esophagus or protect the trachea. The oral airway is
contraindicated in conscious patients and in all patients, even those who are unconscious, who
have an intact gag reflex. Stimulation of the gag reflex may cause vomiting and aspiration.!
!
==================================!
!
A patient has severe facial injuries, inadequate breathing, and copious secretions coming from
the mouth. How should this situation be managed?!
!
A:!
Turn the patient to the side and provide continuous oral suctioning.!
B:!
Insert an oropharyngeal airway and suction until the secretions clear.!
C:!
Alternate suctioning for 15 seconds and ventilations for 2 minutes.!
D:!
Provide artificial ventilations and suction for 30 seconds as needed.!
The correct answer is C;!
!
Reason:!
!
Both inadequate breathing and secretions in the mouth (ie, blood, vomitus, etc) must be
addressed simultaneously. This is best accomplished by suctioning in 15-second increments,
then providing assisted ventilations for 2 minutes. This pattern must be continued until the
airway is clear of secretions or the airway has been definitively secured (ie, endotracheal
intubation). Oral suctioning should not exceed 15 seconds in the adult. The insertion of an
airway adjunct should not occur until the airway is clear of secretions or potential obstructions.!
!
==================================!
!
You are ventilating an apneic adult with a bag-mask device and high-flow oxygen. Her pulse rate
is 130 beats/min and she has cyanosis to her face and chest. The MOST reliable indicator of
adequately performed ventilations in this patient is:!
!
A:!
B:!
decreased compliance with each ventilation.!
slight dissipation of her cyanosis.!
C:!
a decrease in her heart rate to 90 beats/min.!
D:!
noted abdominal rise with each ventilation.!
The correct answer is C;!
!
Reason:!
!
Signs of adequate positive-pressure ventilation include an improvement in heart rate, a marked
improvement in skin color, a ventilation rate that is appropriate for the patient’s age, and the
presence of visible chest rise with each ventilation. In the adult, tachycardia is a compensatory
response to hypoxemia. Adequate ventilation with high-flow oxygen increases the oxygen
content of the blood; as a result, the body’s need to compensate decreases, which manifests as
a decrease in heart rate. Decreased compliance (increased resistance) during ventilations
indicates that the airway is blocked or that the patient’s lungs are difficult to ventilate (ie,
asthma, COPD, CHF). The presence of abdominal rise during positive-pressure ventilation
indicates that more air is going into the stomach than into the lungs.!
!
==================================!
!
All of the following would cause an increased level of carbon dioxide in the arterial blood,
EXCEPT:!
!
A:!
short exhalation phase.!
B:!
reduced tidal volume.!
C:!
deep, rapid breathing.!
D:!
slow, shallow breathing.!
The correct answer is C;!
!
Reason:!
!
Adequate oxygen intake and carbon dioxide elimination require a patent airway and adequate
breathing. The level of carbon dioxide in arterial blood can rise for a number of reasons.
Reduced tidal volume (shallow depth of breathing) results in insufficient oxygen intake and
decreased carbon dioxide elimination. A patient who is breathing slowly (bradypnea) will also
experience a decrease in oxygen intake and reduced carbon dioxide elimination. If exhalation is
impaired, the body will not eliminate adequate carbon dioxide; therefore, it will accumulate in
arterial blood. Deep, rapid breathing (hyperventilation), however, would likely increase carbon
dioxide elimination from the body, thus lowering the carbon dioxide content of arterial blood.!
!
==================================!
!
The respiratory system functions by:!
!
A:!
bringing oxygen into the lungs and eliminating carbon dioxide.!
B:!
sending messages to the diaphragm that cause it to contract.!
C:!
removing carbon dioxide from the cells and returning it to the lungs.!
D:!
ensuring that adequate oxygen is delivered to the body’s tissues.!
The correct answer is A;!
!
Reason:!
!
The function of the respiratory system is quite simplistic: it brings oxygen into the lungs and
eliminates carbon dioxide. The circulatory system works in conjunction with the respiratory
system by ensuring that oxygen is delivered to the body’s tissues via the bloodstream and that
carbon dioxide is removed from the body’s cells and returned to the lungs via the bloodstream.
The pons and medulla (respiratory centers in the brainstem) regulate breathing by sending
messages to the diaphragm and intercostal muscles (the muscles in between the ribs), causing
them to contract; this is a function of the nervous system.!
!
==================================!
!
You are assessing a 66-year-old man who has emphysema and complains of worsened
shortness of breath. He is confused, has a heart rate of 120 beats/min, and an oxygen
saturation of 89%. Which of the following assessment findings should concern you the MOST?!
!
A:!
Worsened shortness of breath!
B:!
Confusion!
C:!
Low oxygen saturation!
D:!
Tachycardia!
The correct answer is B;!
!
Reason:!
!
All of your assessment findings in this patient are significant. Worsened shortness of breath in a
patient with a preexisting respiratory disease could indicate exacerbation of his or her condition
or a new problem. Tachycardia and a low oxygen saturation (SpO2) are signs of hypoxemia, a
low level of oxygen in arterial blood. Of all the patient’s assessment findings, the fact that he is
confused should concern you the most. A decreased level of consciousness in a patient with
respiratory distress indicates that the brain is not getting enough oxygen and that carbon dioxide
is accumulating in the blood. It is important to recognize the signs of hypoxemia and begin
immediate treatment (eg, high-flow oxygen via a nonrebreathing mask, assisted ventilation) in
order to prevent hypoxia, a dangerous condition in which the body’s cells and tissues do not
receive enough oxygen. Left untreated, hypoxia may cause permanent brain damage or death.!
!
==================================!
!
Which of the following statements regarding artificial ventilation of an apneic patient who has
dentures is correct?!
!
A:!
If a patient’s dentures are loose, the EMT should use the jaw-thrust maneuver to keep
the airway open.!
B:!
Tight-fitting dentures should be left in place because they facilitate the delivery of
adequate tidal volume.!
C:!
Because of the risk of airway obstruction, the EMT should routinely remove a patient’s
dentures.!
D:!
The EMT should not attempt to remove a patient’s dentures because this may cause an
airway obstruction.!
The correct answer is B;!
!
Reason:!
!
Providing artificial ventilation with a bag-mask or pocket face mask device is usually much
easier when dentures can be left in place. Leaving the dentures in place provides “structure” to
the face and will assist you in maintaining a good mask-to-face seal, thus facilitating the delivery
of adequate tidal volume. However, loose dentures make it much more difficult to perform
artificial ventilation by any method and can easily obstruct the airway. Therefore, dentures and
dental appliances that do not stay firmly in place should be removed. When ventilating a patient
who has dentures or a dental appliance, periodically reassess his or her airway to ensure they
remain firmly in place.!
!
==================================!
!
In what position would you expect a patient with severe dyspnea to be in?!
!
A:!
Prone!
B:!
Fowler’s!
C:!
Lateral recumbent!
D:!
Supine!
The correct answer is B;!
!
Reason:!
!
The preferred position of comfort for most patients with respiratory distress is the Fowler’s
position (sitting up). A prone, supine, or lateral recumbent position would make it more difficult
for the patient to breathe. If a patient with severe dyspnea is willing to lie flat, the EMT should
take this as an ominous sign and should be prepared to assist the patient's ventilations.!
!
==================================!
!
What is the function of pulmonary surfactant?!
!
A:!
It carries fresh oxygen from the lungs to the left side of the heart.!
B:!
It facilitates the production of mucous, which is expelled during coughing.!
C:!
It lubricates the alveolar walls and allows them to expand and recoil.!
D:!
It dilates the bronchioles in the lungs and enhances the flow of air.!
The correct answer is C;!
!
Reason:!
!
Surfactant is a lubricant that lines the alveolar walls. It allows them to expand and recoil freely,
thereby allowing for an easy exchange of oxygen and carbon dioxide. Diseases such as
emphysema cause destruction of the alveolar walls and a decrease in pulmonary surfactant.
This makes the normal process of breathing very difficult for these patients. Mucous-producing
cells, called Goblet cells, line the trachea and larger bronchi. Provided the patient has an
effective cough reflex, bacteria and other pathogens can be expelled from the body via the
mucous produced by the Goblet cells.!
!
==================================!
!
Which of the following patients has signs of inadequate breathing?!
!
A:!
A 41-year-old woman with shallow respirations of 14 breaths/min!
B:!
A 60-year-old man with clear and equal breath sounds bilaterally!
C:!
A 50-year-old woman with respirations of 12 breaths/min and pink, dry skin!
D:!
A 30-year-old man with respirations of 12 breaths/min with adequate depth!
The correct answer is A;!
!
Reason:!
!
Although the 41-year-old woman has a respiratory rate that falls within the normal range for an
adult, the depth of her breathing is shallow (reduced tidal volume). Signs of inadequate
breathing in the adult include a slow (less than 12 breaths/min) or fast (greater than 20 breaths/
min) respiratory rate, shallow depth (reduced tidal volume), altered level of consciousness,
tachycardia, an irregular pattern of inhalation and exhalation, diminished breath sounds during
auscultation, and cyanosis. It is important to assess ALL components of a patient's breathing:
rate, regularity, depth, and quality. Do not rely solely on one parameter.!
!
==================================!
!
An unresponsive patient’s respirations are 26 breaths/min and shallow. The MOST appropriate
treatment includes:!
!
A:!
a nonrebreathing mask set at 15 L/min.!
B:!
assisted ventilations with 100% oxygen.!
C:!
a nasal cannula set at 2 to 6 L/min.!
D:!
a simple face mask set at 10 to 12 L/min.!
The correct answer is B;!
!
Reason:!
!
Shallow respirations (reduced tidal volume) at a rate of 26 breaths/min will not provide adequate
minute volume. Therefore, you should assist the patient's ventilations with a bag-mask device
and high-flow oxygen. Passive oxygenation devices (eg, nonrebreathing mask, simple face
mask, nasal cannula) will be of little benefit to a patient with inadequate breathing. The patient
must have adequate tidal volume in order to effectively breath in oxygen from these devices.!
!
==================================!
!
Which of the following processes occurs during inhalation?!
!
A:!
The diaphragm contracts and the intercostal muscles relax.!
B:!
The intercostal muscles relax and the diaphragm descends.!
C:!
The diaphragm descends and the intercostal muscles relax.!
D:!
The intercostal muscles and diaphragm both contract.!
The correct answer is D;!
!
Reason:!
!
During the active process of inhalation, the diaphragm contracts, causing it to descend. This
increases the vertical dimensions of the chest. At the same time, the intercostal muscles
(muscles between the ribs) contract, increasing the horizontal dimensions of the chest. These
two processes cause intrathoracic pressure to fall, and air rushes in to fill the lungs. The
drawing of air into the lungs by the actions of these muscles is called negative-pressure
ventilation.!
!
==================================!
!
In which of the following situations should the jaw-thrust maneuver be used?!
!
A:!
When the mechanism of injury is unclear!
B:!
In any patient who is in cardiac arrest!
C:!
In a patient who is in need of frequent suctioning!
D:!
In a patient with apnea with no signs of trauma!
The correct answer is A;!
!
Reason:!
!
The jaw-thrust maneuver should be used to open the airway any time the mechanism of injury
suggests trauma or when the mechanism of injury is unclear (ie, in a patient who became
unresponsive without witnesses). When performed correctly, the jaw-thrust maneuver maintains
a patent airway without manipulating the spine. It should be noted, however, that if the jawthrust maneuver does not adequately open the patient's airway, the head tilt-chin lift maneuver
should be used.!
!
==================================!
!
A 33-year-old female presents with acute respiratory distress. She is conscious but anxious, and
tells you that she has a history of asthma. She took two puffs of her albuterol inhaler prior to
your arrival, but states that it did not help. Her oxygen saturation reads 89% and you hear
diffuse wheezing while auscultating her lungs. You should:!
!
A:!
give her 100% humidified oxygen to dilate her bronchioles, monitor her oxygen
saturation, and transport her to an appropriate medical facility.!
B:!
administer high-flow oxygen, contact medical control to request permission to assist her
with another albuterol treatment, and prepare for transport.!
C:!
ventilate her with a bag-mask device until her oxygen saturation is at least 94% and
rapidly transport her to the closest appropriate medical facility.!
D:!
assist her with a third albuterol treatment, contact medical control for further advice, give
her high-flow oxygen, and transport her to the hospital.!
The correct answer is B;!
!
Reason:!
!
Despite two albuterol treatments, the patient is still experiencing respiratory distress.
Furthermore, the presence of wheezing indicates continued bronchospasm. After administering
high-flow oxygen via a nonrebreathing mask, you should contact medical control and request
permission to assist the patient with a third albuterol treatment. Drugs such as albuterol
(Proventil, Ventolin) and metaproterenol (Alupent) stimulate beta-2 receptors in the lungs,
resulting in bronchodilation. Up to three bronchodilator treatments are typically given in the
prehospital setting. In most EMS systems, EMTs are not allowed to assist patients with their
medication without medical control authorization. After assisting the patient with a third albuterol
treatment, reassess her breath sounds and oxygen saturation and transport her promptly.!
!
==================================!
!
Which of the following processes occurs during cellular/capillary gas exchange?!
!
A:!
The cells receive carbon dioxide from the capillaries.!
B:!
The cells give up oxygen to the capillaries.!
C:!
The capillaries give up carbon dioxide to the cells.!
D:!
The capillaries give up oxygen to the cells.!
The correct answer is D;!
!
Reason:!
!
At the cellular level, oxygen passes across the capillary bed from the arterioles and into the cell,
which is facilitated by a process called diffusion, in which oxygen (as with any gas) moves from
an area of higher concentration to an area of lower concentration. At the same time, carbon
dioxide crosses the capillary bed and enters the venules, where it is transported back to the
lungs for reoxygenation.!
!
==================================!
!
In which position should you place an uninjured, unresponsive patient with a respiratory rate of
14 breaths/min and adequate tidal volume?!
!
A:!
Supine!
B:!
Lateral recumbent!
C:!
Semi-Fowler’s!
D:!
Full-Fowler’s!
The correct answer is B;!
!
Reason:!
!
The recovery position, which involves placing the patient on his or her side (lateral recumbent),
is used to maintain a patent airway in an unresponsive patient who is not injured AND is
breathing on his or her own with a normal rate and adequate tidal volume (depth of breathing).
Patients who are in shock or require positive-pressure ventilation should be placed in a supine
(on his or her back) position. The semi-Fowler’s position involves placing the patient in a
semisitting position at a 45-degree angle; it is the position of comfort for most patients. The fullFowler’s position involves sitting the patient in an upright position at a 90-degree angle; it is
often used for patients with respiratory distress.!
!
==================================!
!
When attaching an oxygen regulator to a D cylinder and preparing it for use, you should recall
that:!
!
A:!
a pressure-compensated flowmeter should be used when lying the oxygen cylinder
down.!
B:!
oxygen supports combustion and should not be used where sparks are easily
generated.!
C:!
the cylinder should be taken out of service and refilled when it contains less than 750
psi.!
D:!
the cylinder must remain in a standing position at all times or it will not deliver any
oxygen.!
The correct answer is B;!
!
Reason:!
!
Oxygen does not burn or explode; however, it does support combustion. A small spark, even a
lit cigarette, can become a flame in an oxygen-rich atmosphere. Therefore, you must ensure
that the environment in which you will use oxygen is adequately ventilated, especially in
industrial settings where hazardous materials may be present and where sparks are easily
generated. Never leave an oxygen cylinder standing unattended; the cylinder can be knocked
over, injuring the patient or damaging the equipment. The D cylinder (small cylinder carried to
the patient) should be taken out of service and refilled when the pressure inside it falls below
500 psi. The pressure-compensated flowmeter, which contains a ball and float that rises or falls
according to the gas flow, can only be used when an oxygen cylinder is upright, which is why it
is used with on-board oxygen (M cylinder). The Bourdon-gauge flowmeter does not require the
oxygen cylinder to be upright, which is why it is used with D cylinders.!
!
==================================!
!
During your initial attempt to ventilate an unresponsive apneic patient, you meet resistance and
do not see the patient’s chest rise. You should:!
!
A:!
assume that a foreign body is blocking the airway.!
B:!
begin CPR, starting with chest compressions.!
C:!
suction the airway for no longer than 15 seconds.!
D:!
reposition the head and reattempt to ventilate.!
The correct answer is D;!
!
Reason:!
!
If your initial attempt to ventilate a patient is met with resistance and/or does not make the chest
visibly rise, you should reposition the patient’s head and reattempt to ventilate. In many cases,
this simple action will open the airway and enable you to ventilate the patient. However, If both
of your breaths are met with resistance and/or do not make the chest visibly rise, you should
assume that a foreign body is obstructing the airway and begin chest compressions. The airway
should be suctioned if secretions are present in the mouth; if oral secretions are not present, do
not suction.!
!
==================================!
!
To ensure you deliver the highest concentration of oxygen with a nonrebreathing mask, you
should:!
!
A:!
cover the flapper valves on the oxygen mask.!
B:!
securely fasten the mask to the patient’s face.!
C:!
make sure that the reservoir bag is preinflated.!
D:!
set the flow rate to at least 10 to 12 L/min.!
The correct answer is C;!
!
Reason:!
!
To ensure delivery of high-flow (greater than 90%) oxygen to your patient with a nonrebreathing
mask, you must first set the flowmeter to 15 L/min and then preinflate the reservoir bag. When
the patient inhales, pure oxygen is inspired directly from this bag. The valves on the sides of the
mask close during inhalation, which prevents outside carbon dioxide from mixing with the
oxygen in the reservoir; they open during exhalation, which allows the patient to eliminate
carbon dioxide. Following inflation of the reservoir, apply the mask to the patient and ensure that
it is secured so as to prevent as much air leakage as possible.!
!
==================================!
!
Hypoxia is defined as:!
!
A:!
an excess amount of carbon dioxide in arterial blood.!
B:!
decreased oxygen content in arterial blood.!
C:!
an absence of oxygen to the vital body organs.!
D:!
inadequate oxygen to the body’s cells and tissues.!
The correct answer is D;!
!
Reason:!
!
Hypoxia is a dangerous condition in which the body’s cells and tissues do not have enough
oxygen. Hypoxemia is a decreased amount of oxygen in arterial blood. Untreated hypoxemia
will lead to hypoxia. An absence of oxygen to any part of the body is called anoxia. An excess
amount of carbon dioxide in arterial blood is called hypercarbia. If the body cannot bring in
enough oxygen, it is also unable to eliminate carbon dioxide from the blood; therefore,
hypoxemia and hypercarbia occur together.!
!
==================================!
!
Oxygen that is administered through a nasal cannula would be of LEAST benefit to a patient
who:!
!
A:!
has COPD and an oxygen saturation of 94%.!
B:!
is in need of long-term oxygen therapy.!
C:!
breathes through his or her mouth.!
D:!
is breathing greater than 12 times per minute.!
The correct answer is C;!
!
Reason:!
!
A patient who breathes through the mouth or has a nasal obstruction will get little or no benefit
from a nasal cannula. Many patients with COPD (eg, emphysema, chronic bronchitis) require
long-term, low-flow oxygen therapy; the nasal cannula is ideal in this situation. Considering their
chronic respiratory problem, an oxygen saturation of 94% in a COPD patient is good; in fact,
many COPD patients maintain an oxygen saturation lower than 94%, even with supplemental
oxygen. A nasal cannula is appropriate to use in patients breathing greater than 12 times per
minute, provided they have adequate tidal volume and are not significantly hypoxemic.
Regardless of the oxygen delivery device used, you should maintain a patient's oxygen
saturation at greater than 94%.!
!
==================================!
!
If an adult patient presents with a respiratory rate of 26 breaths/min, your initial action should be
to:!
!
A:!
begin assisting his ventilations with a bag-mask device.!
B:!
apply oxygen via a nonrebreathing mask and take his vital signs.!
C:!
evaluate his mental status and the depth of his respirations.!
D:!
apply the pulse oximeter and assess his oxygen saturation.!
The correct answer is C;!
!
Reason:!
!
The normal respiratory rate for an adult at rest is 12 to 20 breaths/min. If a patient presents with
a respiratory rate outside of the normal range, you should immediately assess him or her for
other signs of inadequate breathing, such as a decreased level of consciousness, shallow
breathing (reduced tidal volume), brief inhalations followed by prolonged exhalations, and
cyanosis. If the patient is conscious, alert, and has adequate tidal volume (eg, his or her chest
rises adequately with each breath), supplemental oxygen via nonrebreathing mask or nasal
cannula would be appropriate, depending on his or her chief complaint and oxygen saturation.
However, if the patient’s mental status is decreased and his or her tidal volume is reduced (eg,
shallow breathing), some form of positive-pressure ventilation should be initiated (eg, bag-mask
or pocket face mask ventilations). It is important to note that breathing adequacy is not
determined solely by the patient’s respiratory rate; you must assess all aspects of breathing
(rate, regularity, depth) as well as the patient’s mental status. A patient can be breathing at a
“normal” rate; however, if his or her tidal volume is reduced, minute volume will decrease and
some form of positive-pressure ventilation may be required.!
!
==================================!
!
Which of the following patients is the BEST candidate for an oropharyngeal airway?!
!
A:!
B:!
C:!
D:!
An unresponsive patient with uncontrolled oropharyngeal bleeding!
An unresponsive trauma patient with blood draining from the nose!
Any patient that you suspect of being acutely hypoxemic!
A semiconscious patient who ingested a large quantity of aspirin!
The correct answer is B;!
!
Reason:!
!
The oropharyngeal airway is used to keep the tongue off of the posterior pharynx and is
indicated for unresponsive patients without a gag reflex. If an unresponsive patient has severe,
uncontrolled oropharyngeal bleeding, your priority is to suction his or her airway in order to
prevent aspiration and transport rapidly. Semiconscious patients typically have a gag reflex,
although it may be somewhat depressed. Oxygen should be administered to any patient with
suspected hypoxemia.!
!
==================================!
!
Which of the following ventilation techniques will enable you to provide the greatest tidal volume
AND allow you to effectively assess lung compliance?!
!
A:!
One-rescuer mouth-to-mask ventilation!
B:!
One-rescuer bag-mask ventilation!
C:!
One-rescuer demand valve ventilation!
D:!
Flow-restricted, oxygen-powered ventilation!
The correct answer is A;!
!
Reason:!
!
Because the EMT uses both of his or her hands to obtain a mask seal, the one-rescuer mouthto-mask ventilation technique will provide the greatest tidal volume compared to the other
methods listed. Furthermore, lung compliance, the ability of the lungs to expand when
ventilated, can be effectively assessed because air is directly blown into the patient’s lungs from
the EMT’s mouth. The one-rescuer bag-mask ventilation technique may allow the EMT to get a
sense of lung compliance; however, because maintaining an adequate mask-to-face seal is
often difficult, lesser tidal volume can be given relative to the mouth-to-mask technique. The
flow-restricted, oxygen-powered ventilation device (eg, manually triggered ventilator, demand
valve) provides excellent tidal volume; however, because it is a mechanical device, it does not
allow the EMT to assess lung compliance.!
!
==================================!
!
If the level of carbon dioxide in the arterial blood increases:!
!
A:!
the respiratory rate and depth decrease.!
B:!
the respiratory rate slows significantly.!
C:!
a reduction in tidal volume will occur.!
D:!
the respiratory rate and depth increase.!
The correct answer is D;!
!
Reason:!
!
Special receptors, called chemoreceptors, sense the levels of oxygen and carbon dioxide in the
arterial blood. The central chemoreceptors are located in the brain; the peripheral
chemoreceptors are located in the aorta and carotid arteries. The level of carbon dioxide in the
arterial blood stimulates the healthy patient to breathe (primary respiratory drive). If the carbon
dioxide level rises above normal, the chemoreceptors send messages to respiratory centers in
the brain, resulting in an increase in respiratory rate and depth (tidal volume). Conversely, if the
level of carbon dioxide is too low, respiratory rate and depth decrease accordingly.!
!
==================================!
!
A patient overdosed on several drugs and is unresponsive with shallow breathing and facial
cyanosis. As you continue your assessment, the patient suddenly vomits. You should: !
!
A:!
begin assisting his ventilations.!
B:!
suction his oropharynx at once.!
C:!
insert an oropharyngeal airway.!
D:!
turn the patient onto his side.!
The correct answer is D;!
!
Reason:!
!
The patient’s airway must be clear of foreign bodies or secretions before it can be assessed or
managed. If the patient begins to vomit, he must first be rolled onto his side to allow for drainage
of the vomitus. Use suction to remove secretions after you have positioned him on his side.
After the airway is clear, you should insert an appropriate airway adjunct (oral or nasal airway)
and ensure adequate ventilation and oxygenation. In this patient, this involves assisting his
ventilations with a bag-mask device.!
!
==================================!
!
When ventilating an apneic patient with a pocket mask device, each breath should be delivered
over:!
!
A:!
4 seconds.!
B:!
1 second.!
C:!
3 seconds.!
D:!
2 seconds.!
You selected A; The correct answer is B;!
!
Reason:!
!
When ventilating any apneic patient, each breath should be delivered over a period of 1 second
—just enough to produce visible chest rise. Excessive ventilation duration and/or volume
increases the likelihood of gastric distention—especially if the patient's airway is not secured
with an advanced device (ie, ET tube, multilumen airway, supraglottic airway)—and may result
in increased intrathoracic pressure, decreased venous return to the heart, and decreased
cardiac output.!
!
==================================!
!
You are ventilating an apneic 50-year-old woman with a bag-mask device. After squeezing the
bag and noting visible chest rise, you should:!
!
A:!
reopen the airway and ventilate again.!
B:!
allow the patient to completely exhale.!
C:!
suction the airway for up to 15 seconds.!
D:!
squeeze the bag again in 3 seconds.!
The correct answer is B;!
!
Reason:!
!
When ventilating an apneic patient, it is important to allow for complete exhalation. To do this,
deliver each breath over 1 second, just enough to make the chest visibly rise, and then deliver
the next breath 5 to 6 seconds later (3 to 5 seconds later for infants and children). Failure to
allow for complete exhalation may cause the patient to retain carbon dioxide (carbon dioxide
elimination occurs during exhalation) and may also impair venous return to the heart secondary
to hyperinflation of the lungs. If you observe visible chest rise when you ventilate the patient,
there is no need to reopen the airway. Suction the airway only if secretions are present.!
!
==================================!
!
Patients with a hypoxic drive:!
!
A:!
may hypoventilate if given low concentrations of oxygen.!
B:!
are accustomed to low levels of carbon dioxide in the blood.!
C:!
are stimulated to breathe by low oxygen levels in the blood.!
D:!
rarely become cyanotic because of high blood oxygen levels.!
The correct answer is C;!
!
Reason:!
!
Patients with chronic respiratory diseases (eg, emphysema) maintain decreased levels of
oxygen and increased levels of carbon dioxide in the blood. The sensors in the brain become
accustomed to this. Unlike a healthy person, whose primary respiratory drive is influenced by
increasing carbon dioxide levels in the blood, the primary respiratory drive of a patient with a
chronic respiratory disease is influenced by low levels of oxygen in the blood (hypoxic drive).
Cyanosis is common due to chronic hypoxemia. Some patients with the hypoxic drive may
hypoventilate if given high concentrations of supplemental oxygen, although this is highly
uncommon. High-flow supplemental oxygen may fool the brain into thinking the body has
sufficient oxygen, causing it to send fewer signals to the diaphragm and intercostal muscles.!
!
==================================!
!
An unresponsive man has shallow, gurgling respirations at a rate of 8 breaths/min. Initial
treatment should include:!
!
A:!
B:!
C:!
oxygen via nonrebreathing mask.!
suctioning of the oropharynx.!
oropharyngeal airway insertion.!
D:!
positive-pressure ventilations.!
The correct answer is B;!
!
Reason:!
!
Before breathing can be assessed, let alone managed, the airway must be cleared of any and
all secretions. When you hear gurgling respirations, you should provide immediate suctioning of
the oropharynx for up to 15 seconds. After the airway is clear, insert an oral or nasal airway and
begin assisting his ventilations. Shallow respirations at a rate of 8 breaths/min will not produce
adequate minute volume and will require ventilatory assistance. If the patient is continuously
producing oral secretions, you should suction his airway for 15 seconds and then ventilate him
for 2 minutes. Continue this alternating pattern until his airway is clear of secretions or an
advanced airway device (ie, ET tube, multilumen airway, supraglottic airway) has been inserted.!
!
==================================!
!
You are assessing an elderly man with respiratory distress. He is coughing up bloody sputum
and has an oxygen saturation of 85%. You auscultate his breath sounds and hear coarse
crackles in all lung fields. This patient MOST likely has:!
!
A:!
congestive heart failure.!
B:!
acute onset emphysema.!
C:!
severe bacterial pneumonia.!
D:!
decompensated asthma.!
The correct answer is A;!
!
Reason:!
!
This patient’s signs and symptoms are classic for left-sided congestive heart failure and
pulmonary edema. As the left side of the heart weakens, in which case it can no longer
effectively pump blood, blood backs up into the lungs, resulting in pulmonary edema. As
pulmonary edema gets worse, the patient begins coughing up pink, frothy sputum (hemoptysis).
The presence of fluid in the lungs impairs the exchange of oxygen and carbon dioxide, resulting
in hypoxemia and a low oxygen saturation (SpO2). Auscultation of the patient’s lungs often
reveals coarse crackles, which indicates the presence of fluid. Emphysema is a chronic
respiratory disease, not an acute one. Furthermore, hemoptysis is not a common finding with
emphysema. Likewise, patients with decompensated asthma often have markedly diminished
lung sounds owing to severe bronchospasm; hemoptysis and crackles are not common.
Bacterial pneumonia can cause respiratory distress; however, it usually presents with fever and
diminished breath sounds to a localized area of a lung (for example, the left lower lobe).!
!
==================================!
!
An elderly man is found lying unresponsive next to his bed. The patient's wife did not witness
the events that led to his unresponsiveness. You should:!
!
A:!
B:!
C:!
grasp the angles of the lower jaw and lift.!
apply 100% supplemental oxygen.!
assess the patient’s respirations.!
D:!
tilt the head back and lift up the chin.!
The correct answer is A;!
!
Reason:!
!
Because this patient was found unresponsive next to the bed and his wife did not witness the
event, you should assume that he fell from the bed and potentially sustained a spinal injury.
Because of the potential trauma, the jaw-thrust maneuver should be used, which involves
grasping the angles of the lower jaw and lifting forward without manipulating the head.
Performing a head tilt-chin lift maneuver could potentially worsen a spinal injury if one exists.
However, it is important to note that the head tilt-chin lift maneuver should be used if the jawthrust maneuver does not adequately open the patient's airway. After the airway has been
opened and suctioned if needed, the patient’s respirations should be assessed and managed
accordingly. This may include applying supplemental oxygen with a nonrebreathing mask or
assisting his ventilations.!
!
==================================!
!
A woman presents with acute shortness of breath. Her breathing appears labored and her skin
is pale. You should:!
!
A:!
ensure that her oxygen saturation does not fall below 85 percent.!
B:!
administer high-flow oxygen and assess the quality of her breathing.!
C:!
deliver humidified oxygen and administer an inhaled bronchodilator.!
D:!
place her supine and assist her ventilations with a bag-mask device.!
The correct answer is B;!
!
Reason:!
!
Patients with acute respiratory distress and labored breathing need high-flow oxygen. You
should then assess the patient for signs of inadequate ventilation and provide ventilatory
assistance if needed. Patients with labored breathing will probably not allow you to place them
in a supine position as this will make it more difficult for them to breathe. An inhaled
bronchodilator is indicated if you hear wheezing when auscultating the patient's lung sounds,
which you have not done at this point. You should administer oxygen in a concentration
sufficient to maintain an oxygen saturation that is equal to or greater than 94 percent.!
!
!
!
!